SlideShare ist ein Scribd-Unternehmen logo
1 von 42
NAME :AMEER HAMZA
CLASS# 01
DVM FINAL YEAR
(SEMESTER:09)
SEC:A
SESSION:2016-2020
MEDICINE CLINIC 3
ASSIGNMENT SUBMITTED TO :
DR HAQ AMAN ULLAH
Assignment No. 1
In the field you face an outbreakof abortionin
goats during scarcityof green fodder. What
causes of abortionyou will consider, explain
etiology, pathogenesis,clinicalfindings,
treatment and preventionof each cause.
CAUSES OF ABORTION IN GOATS DURING SCARCITY OF GREEN
FODDER :
1. Enterotoxemia
2. Severe stress
3. Exposure to toxic plants, such as broomweed broomweed, turpentine weed
4. Angora abortion syndrome
5. Micronutrient deficiencies copper, selenium, magnesium, vitamin A,E .
6. Bluetongue virus
7. Listeriosis (Circling disease)
8. Johne’s disease
9. Toxoplasmosis
ENTEROTOXEMIA
ETIOLOGY:
It’s a frequently severe disease of goats of all ages. It is caused by two
strains of bacteria called Clostridium perfringens – the strains are termed
types C and D(Pulpy kidney disease, Overeating disease). These bacteria are
normally found in low numbers in the gastrointestinal tract of goats.
These organisms are normally “laying low” in the small and large intestine – that is,
they are present in relatively low numbers and appear to be in a relatively quiescent
state in the normal, healthy animal.
Most commonly, the change that triggers disease is an increase in the amount of grain,
protein supplement, milk or milk replacer (for kids), or grass that the sheep or goat is
ingesting. Collectively, these feeds are rich in starch, sugar, or
protein.
PATHOGENESIS :
Sudden ingestion of readily fermentable, carbohydrate-rich feeds permits
more undigested starch to pass through the rumen to the abomasum and intestine
where it serves as a nutrient substrate for rapid proliferation of the organism.
Excess carbohydrate intake may also predispose to reduced motility. This
proliferation of C. perfringens type D in conjunction with reduced peristalsis enhances
the concentration and pathogenic potential of the epsilon toxin produced by the
organism. Epsilon toxin, after being converted from the prototoxin by intestinal
trypsin, increases vascular permeability in the gut, thereby facilitating its own
absorption into the blood stream. A generalized toxemia ensues. The toxin is
necrotizing, and specififi cally neurotoxic. Death is attributable to damage of vital
neurons, generalized toxemia, and shock.
Kids showed more prominent diarrhea and abdominal discomfort with fewer
neurologic signs preceding death.
At necropsy, intestinal lesions in kids, a severe necrotizing colitis was seen grossly
and confifirmed histologically.
CLINICAL SIGNS :
Three distinct clinical forms of enterotoxemia are recognized in goats, namely
peracute, acute, and chronic.
Peracute form:
occurs more frequently in young goats than adults. The clinical course is usually less
than twenty-four hours and may go unnoticed. Finding one or more dead animals is
often indication of peracute enterotoxemia in a herd.
Clinical signs include a sudden loss of appetite, profound depression, marked
abdominal discomfort manifested by arching of the back and kicking at
the belly, loud and painful screaming, and profuse watery diarrhea containing blood
and shreds of mucus. Fevers of 105°F (40.5°C) are recorded. Affected goats
quickly become weak and recumbent. They may show paddling or convulsions, but
frequently they just lapse into a coma without excitatory signs. Death ensues
within hours. Recoveries are rare, even with treatment.
Acute form:
similar clinical signs are seen but with less severity. Abdominal pain and screaming
may be absent or reduced. Feces may first turn pasty or soft,
but then become watery. The clinical course lasts from three to four days. Severe
dehydration and acidosis, caused by the profuse diarrhea, become complicating
factors in these cases. Spontaneous recoveries may
occur, but most animals die if not treated. The acute form more often affects mature
goats. It can occur in herds with a solid history of vaccination against C.
perfringens type D, so enterotoxemia should not be ruled out on the basis of previous
vaccination.
Chronic form:
intermittent, recurring bouts of illness are observed over several weeks. Mature
animals are usually affected. These goats are dull and listless
with reduced appetite and milk production if lactating. There is progressive weight
loss with intermittent episodes of pasty or loose feces. Abortion occur in pregnant
animals.
Treatment
The prognosis for recovery is guarded in caprine enterotoxemia, even with treatment.
1. Antibiotics like penicillin ,amoxicillin,ampicillin etc for 5 days as per
recommended dose .
2. Intravenous fluid therapy providing mixed electrolyte solutions with bicarbonate is
indicated in peracute and acute cases to counter shock, dehydration, and acidosis.
3.Nonsteroidal anti-inflammatory drugs such as flunixin meglumine (1.1-2.2 mg/kg
I/m every twelve hours) may be helpful in stabilizing animals in toxemic shock and
will alleviate pain.
4.Commercially available type C and D antitoxins should be administered
parenterally, preferably intravenously, in severe cases. Though recommended
prophylactic doses are usually around 5 ml, therapeutic doses up to 100 ml have been
administered. Because the antitoxin products are relatively expensive, a minimum
effective dose is desirable. In the chronic form of the disease, 2 doses of 20 ml of
antitoxin given four days apart was reported to be a reasonably effective treatment
Prevention
Prevention of enterotoxemia is far more likely to be successful than trying to treat the
disease.
1. Vaccination
For sheep and goats, there are multiple vaccines available that induce immunity to the
toxins generated by Clostridium perfringens types C and D.
2. Feeding Strategies
Smart feeding strategies i.e always make feed changes slowly
STRESS
A stress or stressor is a force external to a system that acts to displace the system. A
stress condition can be quantified and applied equally across animals.
ETIOLOGY:
The environmental comfort zone for goats is between 0-30C. Above 30C goats may
begin to experience mild heat stress, especially when humidity cranks up the heat
index. As heat and humidity climb, goats can have serious problems with thermal
stress.
High temperatures affect body function in many ways. The hypothalamus, lying at the
base of the brain, is in charge of balancing the body’s heat loss and gain by regulating
respiration, skin temperature, sweating and muscle tone.When the weather warms up,
animals eat less in an involuntary effort to reduce body heat, feed and water
consumption go down. Animals may reduce water intake however they need water to
help keep them cooler. Weather and other factors can combine to get any goat, but
some are more susceptible to overheating. Overweight goats cannot exchange heat
efficiently. Selenium deficiency may exacerbate heat stress due to marginal muscle
tone. Unventilated confinement, such as being locked in a poorly ventilated barn,
crate or vehicle, can be a serious threat to an animal’s life in a very short time.
PATHOGENESIS:
Nutritional,heat stress negatively affects the ability of an animal to become pregnant
through many mechanisms affecting fertilization, follicular development and early
embryonic development. Heat stress during pregnancy slows down the growth of the
foetus, which was attributed to the decreased uterine blood supply, which hampers
supply of nutrients and hormones to the conceptus . Slow growing embryos fail to
signal pregnancy to the maternal organism in due time. Therefore, the endometrial
prostaglandin F2alpha (PGF2α) secretion tends to increase during heat stress and
trigger luteolysis, thereby threatening the maintenance of pregnancy . The placental
weight and hormonal secretions are reduced and the vascular resistance is increased
during heat stress, which further affects the reduction in perfusion of nutrients to the
foetus .
CLINICAL SIGNS :
1. Panting rapidly. Animals don’t sweat, instead they pant to relieve themselves of
heat. An open mouth and foam around mouth means they are extremely hot.
2. High body temp, anything over 103.2 in hot weather means they are too hot and
can’t cool of.
3. Droopy ears, sunken eyes, lack of interest in food/water/herd.
4. Stretched out on ground and does not make an attempt to get up or moved when
approached.
TREATMENT:
 FLUID THERAPY I.e NORMAL SALINE + VIT B COMPLEX
 MINERALS PROVISION I.E ORS etc
 To sustain pregnancy progesterone drugs like
GLOBINAN,PROGESTERONE,PROLUTON DEPOT can be used.
PREVENTION:
 Ensure there is unlimited access to clean, cool water for all animals.
 Reduce stocking densities.
 shade that encourages air flow
 Temporarily reduce or cease feeding of concentrate and consider a higher
roughage proportion in ration until other emergency measures are
implemented.
Exposure to toxic plants
Snakeweed, broomweed, turpentine weed.
ETIOLOGY:
Botanic Name Gutierrezia sarothrae
Plant Family Asteraceae
Habitat Dry plains and foothills at altitudes from 4000-10000 feet.
Animals Affected
Horses, cattle, sheep, goats
Toxic Principle
Saponins are believed to be the toxic component of
snakeweed. Both the green and dried plant are toxic,
although there appears to be considerable variability in
toxicity.
Description
Perennial which is shrubby or woody only at the base,
attaining 18-24 inches in height. The stems are branching,
the leaves are linear and glabrous. The heads are many,
usually in clusters at the ends of branches. A given head
will have no more than 3-8 ray flowers and 3-8 disc
flowers. The flowers are yellow with the disc flowers
usually perfect. The corollas are 5-lobed and the pappus is
composed of several to many oblong scales. Often forms
dense stands especially in overgrazed rangeland.
CLINICAL SIGNS:
Gastrointestinal
Diarrhea followed by constipation.
Musculoskeletal
Loss of weight
Congenital Defects
Abortions. Calves may be born alive and weak, and may
die after a few days.
Reproductive System
Abortions, retained fetal membranes. May also decrease
male fertility.
Hepatic System
Liver necrosis and degeneration may be seen at
postmortem examination.
Diagnosis
Abortions, liver necrosis, access to snakeweed.
Treatment
There is no specific treatment. Affected animals should be
removed from the snakeweed and giving supportive
care.( MULTIVITAMINS ,ANTIHISTAMINES ,FLUID
THERAPY)
Enzootic Abortion(Chlamydiosis) /Angora
Abortion Syndrome
ETIOLOGY:
is a contagious disease in Angora goats caused by Chlamydia psittaci. Outbreaks are
usually seen a year or two after new goats were introduced onto a farm. The abortions
are more likely to be seen under intensive rather than a veld grazing environment.
Flocks infected for the first time may have up to 70% abortions. These can occur as
early as 3 months into pregnancy where the foetus may be resorbed instead of being
expelled.
Small weak kids may be born that die soon after birth. Kids can survive and carry the
disease. The ewe at no time becomes ill although Chlamydia can be the cause of
infectious opthalmia. It is unlikely that the same ewe will abort again due to
Chlamydia and hence the reason why that after the initial outbreak abortions are then
often seen in maiden ewes on a contaminated farm.
PATHOGENESIS:
The organisms localise in the epithelial cells of the mucosa of the small intestine and
then spread via the blood and lymph vessels. When the organism invades the placenta
the damage can result in embryonal death, resorption, mummification or abortion.
Less extensive damage results in premature or weak full-term kids.
Chlamydia is usually spread during kidding and is transmitted when goat ewes are
grazing plant material that has been contaminated by afterbirth or uterine
fluid. Venereal transmission is also possible when rams with infected accessory sex
glands transmit C. psittaci in their semen. The infection can remain dormant (carrier
state) in the ewe kid until she aborts for the first time. The disease may also flare up
when carrier animals become stressed.
Clinical signs:
 Placentitis, foetal death and abortion
 Enteritis (diarrhoea)
 Pneumonia
 Polyserositis , polyarthritis
 Lymphadenopathy
 Keratoconjunctivitis
 Meningoencephalitis
 Epididymitis, orchitis, seminal vesiculitis
TREATMENT:
1- Tetracycline antibiotic for 5 days
2- NSAID eg flunixin meglumin @1.1-2.2mg/kg I/M for 3 days
3- EYE OINTMENT eg. Polyfax eye oitment,tobramycin (tobradex eye drops) for 5
days.
4- FLUID THERAPY (RINGER LACTATE@150-200ml bid)
Prevention:
Oil-based, inactivated vaccine is produced by Onderstepoort Biological Products
(OBP) to prevent abortions caused by chlamydial infection. It is essential to vaccinate
before the breeding season, because the vaccine will not give protection against
abortion once the foetus has been infected.(4 to 6 weeks before the breeding season)
VITAMINS
The fat-soluble vitamins are important in goat nutrition, as they are in the feeding of
other domestic animals and humans. Water-soluble vitamins other than thiamine and
niacin are generally ignored when formulating caprine rations.
Vitamin A
Vitamin A is a nearly colorless,fat soluble, long chain unsaturated alcohol with five
double bonds.
Beta-carotene is the standard dietary precursor of vitamin A, although some other
carotenoids also have biological activity.
Currently recommended conversions are 671 IU of vitamin A/mg of beta carotene
and 436 IU/mg of other common carotenoids such as cryptoxanthine in yellow corn.
As little as 10% of dietary beta-carotene escapes degradation in the rumen.
Absorption, metabolism, and storage of the remainder require a healthy digestive
epithelium and liver
Vitamin A can also be expressed as retinol equivalents (RE), where 1 IU = 0.3 μg of
retinol.
The biopotency of 1 RE for goats is expressed as the vitamin A activity of 1 μg of
all-trans retinol, 5 μg of all-trans beta carotene, or 7.6 μg of other carotenoids with
vitamin A activity (NRC 2007).
Signs of Defificiency
Poor appetite
weight loss,
unthrifty appearance
poor hair coat,
night blindness
thick nasal discharge
in young animals,diarrheal and respiratory disease and to parasitism.
To further compound the problem, kids with coccidiosis have an
increased vitamin A requirement because of impaired absorption.
Adult goats may have a decreased fertility rate, abortion occurs (related to inadequate
steroid hormone synthesis)
in addition to an increased susceptibility to disease.
Vitamin A deficiency may promote desquamation of urinary epithelium and nidus
formation with subsequent urolithiasis.
PATHOGENESIS:
Carotenoids are the principal source of vit A for small ruminants.The green part of
plants are typically rich in carotene. The likelihood of a deficiency is minimal if small
ruminants have access to modest amounts of green fodder.
Alternatively,low carotene intake by grazing livestock is common in geographical
locations that experience prolonged dry season & under condition of winter feeding
may not occur more than 3-4 months.The circumstances conducive to vitamin
deficiencies are extended periods of draught,diets primarily composed of grain based
conncentrates mixture low in retinol equivalents neaonatal animals consuming milk
from mothers on a low carotene content are substituted for yellow corn.
The 1st stage was marked by the development of xerophathalmia which start with
corneal opacity followed by ulceration of cornea and ended with the onset of corneal
ulceration.
The 2nd stage of deficiency was associated with a variety of symptoms starting with
abnormal fluctuations in body weight followed by growth retardition and weight loss
At about same time, weakness & incoordination of movement
developed .Thereafter,convulsions were followed by death.Although clinical signs of
deficiency were not observed in a long term Vit A depletion study.
The hypovitaminotic A adult and a higher incidence of squamous metaplasia of the
parotid duct and glands.Goats experiencing Vit A deficiency had reduced feed intakke
and lostbody wt after 25 weeks.
Vit A deficiency also decrease sperm quality,in pregnant goats they may abort or
produce blind ,dead and weak offspering.
Retained placenta case also exists.
Dietary Recommendations and Supplementation
The NRC applies sheep requirements to goats. According to its 2007
recommendations, the daily maintenance requirement is 31.4 RE/kg body weight
or 104.7 IU/kg.
The 50-kg goat at maintenance should receive 1,570 RE or approximately 5,235 IU
of vitamin A/day while the 90-kg goat needs approximately 9,423 IU.
To this is to be added 45.5 RE/kg (152 IU/ kg)/day for late pregnancy and 100
RE/kg (333 IU/ kg) for growing goats.
The NRC 2007 additional requirement for lactation is now 53.5 RE/kg and
dis regards the level of milk production. A simplifified recommendation from the
French is 5,000 IU vitamin A/kg dry matter of feed across the board
Browse and green leafy hays are good sources of vitamin A, while old or weathered
hay is a poor source. After six months of storage, all beta-carotene in hay has been
destroyed.
Vitamin A palmitate is commonly added to mineral mixes and commercial
concentrates. Because vitamin A is fat-soluble and stored in the liver and other body
fat, the adult goat can tolerate several months of low carotene intake without
developing clinical signs of deficiency. Vitamin A toxicity does not occur when
natural diets are fed but could occur with mixing errors. A maximum of 6,000 μg of
retinol/kg bodyweight/day is recommended (NRC 2007).
Colostrum is a very rich source of vitamin A, and before its consumption kids have
minimal stores. When kids receive colostrum from does that are primiparous
(decreased concentration of vitamin A), or only limited amounts of colostrum, an oral
vitamin A palmitate supplement is desirable. Injectable vitamin
A is less valuable because of rapid peroxidation at the injection site.
Vitamin E
Vitamin E (1 mg dl-alpha tocopheryl acetate = 1 IU) is present in colostrum, milk,
and many natural feeds, especially green forage. Ruminants do not synthesize
vitamin E, but require it in their diet . Its main action is as an antioxidant; it stabilizes
polyunsaturated fatty acids, vitamin A, and various hormones and enzymes. Vitamin
E and selenium are closely interrelated; a defifi ciency of one can be at least partially
offset by increasing the intake of the other.
Signs of Deficiency
Nutritional muscular dystrophy (white muscle disease). This is most likely to occur
with feeding of silage or old hay, because the vitamin is lost in storage. Kids may
have muscle disease at birth and be too weak to suckle. Sudden death related to
Zenker’s necrosis of heart muscle or diaphragm may occur, even in kids with normal
selenium status.
Inhalation pneumonia can also result from muscle weakness of the larynx and
pharynx.
Reproduction failure, abortion in doe
Affected kids are sometimes noted to cough or have milk run out the
nose after drinking. Kids may also develop muscle stiffness after exercise.
Adults may show poor uterine involution and retained placenta.
Undesirable milk flavors related to oxidation of milk fat might be expected to occur
with vitamin E/selenium deficiency.
Ventral edema of Angora goats has also been associated with vitamin E deficiency.
Vitamin E is also very important for optimizing immune responses.
Beneficial effects have been demonstrated in ruminants on phagocytosis by
polymorphonuclear cells and on cell mediated immunity.
Obviously, increased incidence or severity of infectious diseases is a very
nonspecifific sign of vitamin E deficiency, but is part of the reason for recent
increases in the dietary recommendation.
Plasma vitamin E concentrations of less than 1.5 μmol/l (65 μg/dl) in preparturient
does and less than 1 μmol/l (43 μg/dl) in suckling kids have been associated with
increased risk of myopathy .
A normal range in goat serum of 60 to 150 μg/dl has been proposed by the Colorado
Veteri nary Diagnostic Laboratory . Note that the blood sample needs to be handled
very carefully (no hemolysis, rapid refrigeration and removal of the plasma or serum
from the erythrocytes) if accurate results are to be obtained. Liver vitamin E
concentration can also be determined and may better reflect the nutritional status of
the animal.
A normal caprine liver concentration of greater than 250 μg/100 g wet weight has
been proposed.
Dietary Recommendations and Supplementation
In the absence of selenium defifi ciency, the daily requirement of vitamin E for
preventing nutritional muscular dystrophy in preruminant lambs and calves,
and presumably, kids was previously given as 0.1 to 0.3 IU/kg bw . This was to be
doubled when feeding milk replacers. Later, it was advised to add 25 to 50 mg of
vitamin E/kg of concen trate for adults and 50 to 100 mg/kg of concentrate for kids.
Note that vitamin E is relatively nontoxic and 75 IU/kg bw daily is presumed safe.
The current recommendation of 10 mg/kg bw daily takes into account
the diverse functions of the vitamin beyond prevention of mypoathy.
Vitamin E is oxidized by iron (or copper) in the feed, and thus diets with high iron
content may have less vitamin E available. Rapidly growing green plants
have a high content of polyunsaturated fatty acids (PUFAs) and animals on lush
pasture also have a higher need for antioxidants such as vitamin E because
of increased incorporation of PUFAs into cell membranes and thus increased
susceptibility to lipid peroxidation .
Oil seeds also contain high concentrations of PUFAs. The vitamin E content of
colostrum depends on the nutrition of the dam during pregnancy. At the end of the
winter, when most goats in temperate climates kid, the hay cut the previous year is
very low in vitamin E.
Supplementation of the pregnant doe is important for optimizing kid health.
Some injectable preparations such as vitamin A and D include vitamin E as an
antioxidant to stabilize the other fat-soluble vitamins; the quantity of vitamin E is
inadequate for therapeutic purposes.
Similarly, some vitamin E/selenium preparations contain relatively little vitamin E.
Injectable preparations of vitamin E alone are available, with labeled
recommendations of 600 to 900 IU to lambs at birth or weaning and 1,200 to 1,500 IU
to ewes prepartum or at lambing.
Magnesium
Magnesium has received far less study than have calcium and phosphorus, although
the metabolism of these minerals is interrelated. Approximately 62% of body
magnesium is deposited in bone, 37% in cells, and 1% in extracellular fluid.
Magnesium is required for many enzyme systems (including those necessary for
energy metabolism and for synthesis of RNA and DNA) and for normal
neuromuscular function.
Signs of Deficiency:
The normal serum magnesium concentration for goats is 2.8 to 3.6 mg/dl .
Hypomagnesemic tetany typically occurs if the serum level drops below 1.1 mg/dl .
Serum calcium may be low because magnesium is required for the release and action
of parathyroid hormone.
A magnesium deficiency may lead to
anorexia
hyperexcitability.
decreased production,
teeth grinding,
salivation,
tetany,
seizures,
recumbency,
ABORTION
Death,
Goats can compensate somewhat for a dietary magnesium defificiency by decreasing
output of both milk and urine.
Dietary Recommendations and Supplementation
The percent magnesium absorption probably varies considerably with the diet and is
not regulated by a hormonal feedback system.
Transit time that is too rapid or a potassium excess may hinder magnesium
absorption.
Grasses growing rapidly in cool, wet weather or after heavy fertilization are often low
in magnesium and high in potassium. Forages containing less than 0.2% Mg on a dry
matter intake (DM) basis have been associated with hypomagnesemic syndromes in
ruminants.
Whole goat milk also occasionally supplies inadequate magnesium, and kids might
develop tetany if not supplemented
For an animal that weighs 60 kg, then, total daily intake of magnesium is 1 g for
maintenance (approximately 0.045 g/kg 0.75), plus 1.5 g for late gestation.
The approximate magnesium supplement needed per kg of milk is 0.7 g.
When additional magnesium is needed in the diet, it is often supplied at MgO,
although MgCO3, MgSO4, and MgCl2 are other, although less palatable or more
laxative possibilities.
On spring pastures, provision of NaCl in loose or block form
helps to counter high dietary potassium and maintain absorption of magnesium from
the rumen.
Copper and Molybdenum
These two minerals are closely interrelated. High molybdenum (above 3 mg/kg) in the
feed results in a relative copper deficiency, probably through formation
of copper-molybdenum complexes in the tissues.
Signs of Deficiency:
nonspecific signs copper deficiency
Inappetence,
poor growth
, weight loss,
decreased milk production
decreased cytochrome oxidase activity.
Anemia occurs because ceruloplasmin is required to mobilize stored iron for
synthesis of hemoglobin and myoglobin.
Decoloration of the hair occurs because a copper-containing enzyme is necessary for
melanin production.
Swayback and enzootic ataxia in kids are related to defective myelination.
Cardiac insufficiency is probably due to a combination of problems, including
inadequate cytochrome oxidase activity and anemia.
Osteoporosis and spontaneous bone fractures are also related to effects on
copper-dependent enzymes.
Abortions and stillbirths also occur.
In addition, copper is required for proper function of the immune system.
Dietary Recommendations and Supplementation
Deficiency symptoms occur when dietary copper is less than 7 mg/kg and
molybdenum is normal.
A suitable level for ration formulation is 10 to 20 mg/kg DM , and it is generally
recommended to keep the Cu : Mo ratio above 2 : 1 and below 10 : 1.
Excessive calcium and sulfur both interfere with copper absorption as does excessive
dietary iron.
With feeding of corn silage or sulfur, dietary copper should be at least 14 mg/kg.
Note that copper oxide is only one-third as digestible as copper sulfate. When the
basal diet is deficient in copper or contains markedly excessive amounts of
molybdenum, copper oxide wires administered orally in a gelatin capsule provide
long-term (six-month) supplementation because they lodge in the abomasum and
release copper slowly .
An injectable supplement containing chelated copper is available in the United
States labeled for goats.
Copper Toxicity Adult goats are not as susceptible to copper toxicity
as sheep , in part because of lower uptake by the liver. Liver copper stores are
approximately ten times lower in normal goats than in sheep and cattle. In a toxicity
study, hepatic copper concentrations were six to nine times higher in three-month-old
lambs than kids.
A deficiency of molybdenum (less than 0.1 mg/kg , then, does not generally induce
copper toxicity but rather interferes with normal growth and fertility . Also, goats can
safely consume a trace mineralized salt preparation formulated for cattle or a horse
grain mix, whereas this is dangerous for sheep.
Young kids, however, are sensitive to increased copper levels in the feed .One
example of copper toxicity was reported in young Angora goat kids receiving milk
replacer formulated for calves (10 mg/kg copper on a DM basis). The kids died of a
copper-associated hemolytic crisis. It is likely that the preruminant kid absorbs copper
more efficiently than does the adult goat. Fatal hepatic necrosis without hemolysis has
also occurred in adult goats fed an improperly formulated mineral mix.
Iodine
In the absence of adequate iodine, the thyroid gland synthesizes an uniodinated
inactive prehormone rather than thyroxine. In response to lower thyroxine levels the
pituitary gland secretes thyroid stimulating hormone (TSH). As a result, the thyroid
gland hypertrophies and produces the clinical condition goiter.
Goats may produce kids with goiter on the same property where sheep and their lambs
remain healthy.
This is because the goats’ browsing habits result in less soil ingestion compared with
the grazing sheep.
Signs of Deficiency
birth of weak or dead kids and a poor hair coat.
Kids may appear “dumb” or unwilling to suckle.
Growth rate of kids is reduced, as is the fertility of does.
Dietary Recommendations and Supplementation
The requirements of ruminants can usually be met by feeding 0.8 mg/kg iodine to
lactating females and 0.5 mg/kg to the remainder of the herd. Cruciferous plants
increase the iodine requirement to 2 mg/kg in the ration dry matter for lactation and
1.3 mg/kg for other animals. Iodized salt is a simple way to prevent defifi ciency, but
it should not be force fed. A maximum tolerable dietary iodine level of 50 mg/kg has
been established for cattle and sheep, with the proviso that the iodine concentration in
the milk of animals on such a diet may be undesirable for humans . Owners should
likewise be discouraged from feeding large quantities of kelp and other concentrated
iodine supplements.
Manganese
Signs of manganese deficiency in goatsinclude
reluctance to walk,
deformed forelimbs (caused by defective cartilage formation),
excessively straight hocks,
reduced fertility (including silent estrus) or abortion in does
Buck kids appear to show a greater depression in growth rate than doe
kids when fed an extremely manganese-deficient diet (1.9 mg/kg).
The recommended dietary level for goats by various authors has ranged from 20 to
120 mg/kg , but 60 mg/kg allows for interference with absorption, as by excess
calcium.
Selenium
Selenium deficiency occurs when the soil in a locality is defifi cient (less than 0.5 mg
Se/kg of soil) and locally harvested feeds are fed (less than 0.1 mg Se/kg
of feed) . Selenium deficiency has occurred in animals and humans in many parts of
the world, including the United States, China, Finland, New Zealand, and
Australia.
Signs of Deficiency:
Many selenium deficiency signs are identical to those of vitamin E deficiency, as
mentioned earlier.
Nutritional muscular dystrophy, which can be caused by either vitamin E or selenium
deficiency,
Experimental selenium deficiency (less than 38 μg/kg DM) has produced lowered
reproductive efficiency (apparent lowered conception rate) and decreased production
of milk, milk fat, and milk protein in the following lactation.
Selenoproteins act as antioxidants and are also involved in the conversion of T4 to T3.
Increased supplementation with vitamin E masks a mild selenium deficiency.
Potential selenium deficiency problems can be identified in clinically normal
animals by evaluating their glutathione peroxidase status, because selenium is
required for GSH-Px formation
selenium deficiency can be suspected if the goat has less than 5 μg selenium/dl blood
(less than 0.05 mg/kg).
Serum selenium concentrations less than 0.05 mg/kg have also been deemed to
indicate deficiency . Liver selenium content of animals that die or are slaughtered can
be used to monitor the selenium status of the herd. Concentrations of 0.25 to 1.20
mg/kg wet weight are adequate, while concentrations of 0.01 to 0.10 mg/kg wet
weight are deficient . Maternal liver stores of selenium are decreased in advanced
pregnancy as transfer to the fetus occurs.
When conversion of units is necessary for interpretation of laboratory reports, it helps
to know that 1 μg selenium/dl is equivalent to 0.127 μmol/l
.
Dietary Recommendations and Supplementation
Selenium should be present in the diet at a minimum of 0.1 mg/kg of feed.
Sodium selenite and sodium selenate are permitted.
Feed mills cannot add more than 0.3 mg/kg selenium to a complete ration for cattle
or sheep or 90 mg/kg to a sheep salt-mineral mix, nor should the maximum daily
intake of added selenium for sheep exceed 0.7 mg/head/day
Supplementation of goat diets with selenium yeast is specifically permitted in the
United States, with up to 0.3 mg/kg added selenium in this form allowed in complete
feeds (FDA 2005). In the European Union, the maximum allowed selenium inclusion
rate in the ruminant diet is 0.568 mg/kg, and feeding selenium yeast at ten times this
level did not result in toxicity .
When the soil, and hence the roughages and grains grown on it, are deficient in
selenium, several methods have been used to improve selenium content of feeds for
goats. One is to use sodium selenate in fertilizer mixes applied to the fields. In
Finland, this practice has increased the selenium content of feeds from 0.02 mg/kg
DM to 0.2 mg/kg DM .
Deficiency symptoms are most likely to develop:
These include shortly before breeding and four and/or six weeks before parturition for
does, twice a year for bucks, and at birth and one month of age for kids.
The dose administered is typically one to two times the labeled sheep dosage, with
kids of normal size receiving the “minimum” dose at birth instead of two weeks
of age. Because injectable selenium preparations available in the United States are not
labeled for goats and are labeled as not for use in pregnant sheep, practitioners should
be cautious about prescribing them for pregnant goats without informed owner
consent.
Selenium Toxicity
There is a relatively narrow margin of safety with selenium; the maximum tolerable
level in the feed of ruminants is currently estimated to be 5 mg/kg.
Certain soils are termed “seleniferous” because of an increased selenium content,
and certain indicator plants require and accumulate increased concentrations of the
mineral.
Some of these plants found in the United States are Stanleya, Haplopappus, and
some species of Astragalus. They are very useful for indicating that the soil is
dangerous, but they are not the only plants that accumulate toxic levels. Most crop
plants,grasses, and weeds can accumulate as much as 50 mg/kg selenium when grown
on seleniferous soils .
. Acute selenium toxicosis (depression and dyspnea) has been produced
experimentally in sheep with injections of 0.4 mg Se/kg bw, and the LD50 in this
study was 0.7 mg Se/kg.
Necropsy lesions included pulmonary edema and myocardial necrosis. Practitioners
must guard against accidentally substituting an injectable selenium product marketed
for adult cattle for the lower concentration calf and sheep product when small kids are
treated. Selenium is less toxic when given orally. Daily oral doses of sodium selenite
at 1 mg/kg body weight/day were nontoxic to growing Nubian goats, whereas a single
dose of 40 mg/kg or two daily doses of 20 mg/kg were rapidly fatal.
Adverse reactions (deaths and abortions) in several flocks of pregnant sheep have led
to the relabeling of injectable selenium in the United States as not for use in pregnant
sheep. These products are not approved for goats, and thus the practitioner who
prescribes injectable selenium for pregnant goats may be at increased risk of legal
action should any adverse reactions or unrelated abortions occur.
PREGNANCYTOXEMIA:
Excessive grain feeding itself can lead to pregnancy toxemia because the goat that
eats inadequate rough age is apt to go off feed at this critical time. When goats
are heavily fed on corn silage in late gestation they become obese. Their level of
ingestion then decreases dramatically before parturition. Rumen acidosis from
energy-rich silage may contribute to the development
of pregnancy toxemia.
Pathogenesis:
Pregnancy toxemia is typically more common than lactational ketosis, and occurs
predominantly in “improved” breeds with high prolififi cacy. It is not a disease
expected to occur in native breeds carrying a single kid under extensive management
conditions. With proper management and nutrition, even the doe carrying quadruplets
can remain clinically healthy, although concentrations of ketone bodies in the blood
can be expected to be higher in those goats carrying large litters.
The developing fetuses depend upon glucose (maternal hepatic gluconeogenesis) for
their energy needs. Ketone bodies and free fatty acids do not cross the placenta in any
substantial quantities (Reid 1968). Insulin levels in the late pregnant doe are
decreased; this spares glucose for fetal needs while at the same
time stimulating lipolysis and gluconeogenesis. Placental lactogen levels are greatly
increased when multiple fetuses are present . Placental
lactogen has growth hormone as well as prolactin activity and is probably crucial to
meeting the metabolic needs of the fetuses at the expense (if necessary) of the dam.
Thus, the late pregnant doe is often subclinically ketotic.
Clinical Signs
The early signs of pregnancy toxemia are vague.
They probably originate from decreased glucose utilization by the doe’s brain.
The goat may be slow to get up or may lie off in a corner.
It eats less and its eyes are dull.
There is often a noticeable subcutaneous edema of the lower limbs.
Teeth grinding and generalized weakness progress to more apparent neurologic
abnormalities (blindness, loss of menace response, stargazing, nystagmus, ataxia,
tremors), then to coma.
The fecal output is reduced to a few small, dry, mucus coated pellets.
As metabolic acidosis develops the animal may breathe more rapidly.
Thus, advanced primary ketosis may be difficult to distinguish from a primary
pneumonia which has caused the goat to go off feed and develop a secondary ketosis.
A careful physical examination is also necessary to identify other problems such as
parasites, lameness, and bad teeth that might have contributed to the animal’s present
state.
In the terminal stages of pregnancy toxemia, the doe becomes recumbent. Death of
the fetuses at this stage releases toxins and hastens the demise of the doe.
Pulse and respiratory rates increase as endotoxic shock develops. The course of the
untreated disease varies from twelve hours to one week.
Goats with pregnancy toxemia that do not die tend to have dystocias and higher kid
mortality.
They do not come to milk well.
Similar problems are noted in obese goats, even when clinical pregnancy toxemia
does not occur; in addition, these animals are at high risk of developing lactational
ketosis.
Laboratory Tests
There are three major ketone bodies produced in the course of this metabolic disease:
beta-hydroxybutyrate (BHB), acetoacetate, and acetone. In the past, these were
sometimes measured together in a poorly defined way and reported as total ketones.
Currently, BHB, which is the most stable ketone in blood and accounts for
approximately 85% of the total ketones in sheep with pregnancy toxemia, receives the
most attention in the laboratory.
Some people can detect an odor of ketones on the breath of ketotic animals.
Others must depend on simple diagnostic reagents or laboratory tests.
In the early stages of pregnancy toxemia, ketone bodies are easily detectable in the
urine. The commonly used test strips and pills containing nitropruside turn purple in
the presence of acetoacetate but react minimally with acetone and BHB.
If the doe has only a trace ketonuria (physiologic when late pregnant with multiple
fetuses), some other cause for its illness should be sought, but supportive treatment to
prevent worsening of the ketosis should be given. Late stages are usually
accompanied by renal failure; marked proteinuria, epithelial casts, and ketonuria are
present. The veterinarian should have a collection cup close at hand whenever
examining a late-pregnant goat.
BHB values less than 1 mmol/l can be considered normal, values of 1.5 to 3 mmol/l
can be considered indicative of severe undernutrition, and animals with
pregnancy toxemia often show a BHB concentration greater than 3 mmol/l.
For conversion to other units, note that BHB in mmol/l × 10.3 = BHB in mg/dl.
Necropsy Findings
The doe that dies of pregnancy toxemia usually has multiple fetuses in the uterus,
unless these were removed just before death. The fetuses may be fresh or
decomposed.
The doe’s liver is enlarged and yellow because of infifi ltration with fat . The doe’s
adrenal glands are enlarged.
The carcass appears dehydrated. If urine remains in the bladder, it shows a strong
ketone reaction.
Treatment
The treatment and prognosis depend on the stage of the disease.
In the earliest clinical form, the goat readily eats offered grain. Its diet should be
improved to include better quality roughage and increased concentrates.
Propylene glycol is given orally by dosing syringe, at the rate of 60 ml two or three
times daily as a glucose precursor. Although some authors suggest as much as 175 to
250 ml of propylene glycol twice a day , this dosage seems excessive and likely to
overwhelm the ability of the rumen flora of an already sick goat to digest it.
A commercial product that contains niacin as well as propylene glycol has been
recommended, or the goat can be injected with enough mixed B vitamins to supply 1
gram of niacin/day.
Calcium borogluconate (60 ml of a 23% to 25% solution) is given subcutaneously to
counteract any concurrent hypocalcemia; approximately 20% of sheep withpregnancy
toxemia are also hypocalcemic.
If the animal is unwilling to eat or to rise, the prognosis is guarded.
Intravenous glucose (25 to 50 grams, preferably as a 5% to 10% solution), mixed B
vitamins, and force feeding are added to the regimen.
If the goat is known to be within one week of its due date, hormonal induction of
parturition with 10 mg of prostaglandin F2 alpha will end the
energy drain to the fetuses.
If the due date is uncertain and the owner desires to save doe and kids, 20 to 25 mg
dexamethasone may be preferred for its gluconeogenic effects and beneficial stimulus
to appetite.
Prevention
Animals must be fed high quality rough age and as much as 500 g of concentrate
daily. Any conditions that disturb the comfort of the goat, such as lack of exercise,
poor ventilation, or drafts, should be corrected. This means that the stall should be dry,
well bedded, and uncrowded. The goats should be let loose for at least two to three
hours per day.
Goats carrying three or more fetuses should receive the best quality roughage
available in addition to adequate concentrate.
When one doe develops pregnancy toxemia, the diet of the rest of the herd must be
evaluated and corrected as necessary. Concentrates should be introduced gradually
and under strict control, to avoid indigestion. It is unrealistic to expect a large herd of
commercial goats to be totally free of pregnancy toxemia.
Routine monitoring of all late-pregnant does for urinary ketones is also unrealistic.
Healthy does carrying large litters can be expected to excrete small quantities of
ketones yet do not need treatment. Drenching with prophylactic propylene glycol (60
ml orally twice a day) is in itself a stress to a goat and should be reserved for those
showing abnormal behavior or diminished appetite.
Bluetongue in Goats
A severe viral disease caused by an orbivirus transmitted mainly by gnats of the genus
Culicoides. Transmission sexually and across the placenta can also occur. Because the
vector is a gnat, the spread of this disease occurs primarily in the late summer and fall.
The virus is endemic in many areas and cattle and wild ruminants, or white-tailed deer,
act as reservoirs.
Goats are commonly infected with the virus but rarely show any signs of clinical
disease; it is a self-limiting disease in goats.
Symptoms:
transient fever
swelling of the face, muzzle, and ears;
large amount of nasal discharge which may cause crusting around the nose;
oral mucus membranes become dark pink
small hemorrhages and ulcers may form on the roof and corners of the mouth.
The tongue may become cyanotic (blue) but not as common as the name indicates.
Laminitis can develop caused by inflammation of the coronary band and tissues of the
foot to the point that some animals may slough their hooves.
Diarrhea and wool-break will also occur in infected animals.
Bluetongue virus will cause abortions, stillbirths and weak lambs.
Diagnosis:
By the presence of clinical signs similar to those reported in sheep have been
documented in goats.
Treatment:
Minimize animal stress
antibiotic treatment for secondary infections I.e amoxicillin for 3 days.
Prevention:
Controlling breeding areas for biting gnats. Keeping animals away from areas where
biting gnats are present. Vaccine is available for sheep.
Listeriosis
Listeriosis is an important infectious disease of goats most commonly associated with
neurologic disease, but also capable of causing septicemia and abortion. The organism
can be shed in the milk of healthy-appearing carrier goats as well as in the milk of
sick goats. The zoonotic potential of listeriosis from milk and dairy products is a
growing concern
.
Etiology
Listeria monocytogenes is a motile, aerobic and facultative anaerobic, small,
Gram-positive rod. It produces a narrow zone of beta hemolysis on blood agar.
It is capable of growth over a wide pH range of 5.5 to 9.6 and a temperature range of
37.5°F to 113°F (3°C to 45°C), but optimal growth occurs at pH 7 to 7.2 and a
temperature range of 68°F to 104°F (20°C to 40°C).
Isolation of the organism from tissues and organic materials such as animal feeds
can be difficult, so dispersion of tissues in a blender, cold enrichment of samples,
subculturing from tryptose phosphate enrichment broth, and use of selective media
such as trypaflavine nalidixic acid serum agar have been recommended.
L. ivanovii is considered non-pathogenic to humans but is pathogenic for mice and
has been associated with abortions in sheep and cattle. In one recent report, abortions
were reported in sheep in a mixed sheep and goat flfl ock, but the goats were not
affected. Though easily killed by common disinfectants, L. monocytogenes can
survive in feces, silage, and tissue for fifi ve or more years
There are now sixteen known serotypes with numerous subtypes. Serotype 4,
especially type 4b, and to a lesser extent serotype 1 have been associated with
encephalitis and septicemia in goats. Abortion is associated primarily with serotype
1 .
In one outbreak of listeriosis in goats, the same serotype, 4b, was recovered from
goats with encephalitis and from goats with abortion. It was hypothesized that
transmission in this herd outbreak was by the venereal route. Serotypes 1/2a, 1/2b,
and 4b are the serotypes most commonly isolated from human cases
of listeriosis and from livestock cases.Because L. monocytogenes is widely distributed
in nature, characterization of environmental isolates by serotyping or phylogenetic
analysis is necessary to confirm their association with disease outbreaks.
Epidemiology
As many as forty species of birds and mammals, including humans, can be infected
with L. monocyto genes and the organism has been isolated on six continents.
Listeriosis is a well known, sporadic clinical problem in intensively managed dairy
goats in North America and Europe. In France, 4.9% of fecal samples from sheep and
goats in ninety-eight flocks yielded L. monocytogenes. A seroepidemio logic study
in Spain identififi ed infection in 5% of goat herds tested.
Caprine listriosis has also been reported from Japan , South Africa , Australia ),
India , Brazil , and Turkey .
Factors predisposing to clinical listeriosis in goats are similar to those reported for
other farm animals and include sudden changes in weather, feeding regimens, or
general management procedures; confinement in winter, particularly if overcrowded
and with poor sanitation; increased stress from poor nutrition, parasitism, or other
concurrent disease; advanced pregnancy; and the feeding of silage, particularly poor
quality silage. The feeding of silage is often empha sized as a key predisposing
factor in ruminant listerio sis (Morin 2004). However, a history of silage feeding
is not a prerequisite in outbreaks of caprine listeriosis
An increased occurrence of listeriosis in fall and winter has also been observed in
goats, though cases can occur year-round. The disease is most common in
adult goats. Based on experimental challenge studies, goats are more susceptible to L.
monocytogenes infection than sheep.
A survey from Greece based on microbial cultures of brains from animals with
neurologic signs of disease also found that goat herds were affected with encephalitic
listeriosis more frequently than sheep flfl ocks, with serotype 4b being predominant .
The source of infection in herds is not always clear.
Recent studies have identififi ed some differences in the ecology and transmission of
L. monocytogenes on cattle farms as compared to sheep and goat farms in
upstate New York. Cattle farms had a higher level of environmental contamination
with L. monocytogenes than small ruminant farms whether the cattle had a
history of clinical listeriosis (case farms) or not (control farms).
When small ruminant case farms were compared with bovine case farms, isolation of
L. monocytogenes in small ruminant fecal samples was signififi cantly less common
than in bovine fecal samples. However, the organism was signififi cantly more
common in feed samples from small ruminant case farms than from bovine case farms,
indicating that listeriosis on goat and sheep farms is more likely transmitted via feed
than feces. On all farms, soil samples were positive more commonly than were feed
samples, indicating that soil is an important source of feed contamination
with L. monocytogenes.
Direct transmission of L. monocytogenes from animals to humans can occur, it is
uncommon. In such cases human symptoms are usually limited to localized cutaneous
infections. Much more common is the transmission of the infection to humans via
foods of animal origin. Zoonotic infection from goat milk and goat milk products is a
very real concern. Listeria mono cytogenes can be shed in the milk of clinically
affected goats as well as normal-appearing latent carriers.
Shedding is less likely in the encephalitic form of the disease than in the septicemic or
abortion forms. In latent carriers, the intensity of shedding is increased
toward the end of gestation ). Listeria monocytogenes has resisted pasteurization at
143°F (61.7°C) for thirty-five minutes, but is killed by high temperature short-time
pasteurization at 160.9°F (71.6°C) for fifi fteen seconds. The intraleukocytic location
of some of the organisms in milk presumably contributes to this pasteurization
resistance (Blenden et al. 1987). Experimentally, the organism has been reisolated
from semi-soft, aged, goat milk cheeses made from unpasteurized, L.
monocytogenes-inoculated goat milk as long as eighteen weeks after preparation . The
organism has been isolated from retailed, pasteurized, flfl uid goat milk in the UK
(Roy 1988). In a study in Sri Lanka, L. monocytogenes was recovered from raw goat
milk, standard pasteurized milk, and cheese, but not from sterilized milk, ultra-high
tem perature (UHT) milk, yogurt, or curd.
Pathogenesis
In the encephalitic form of listeriosis, the organism gains entrance to nerve endings in
the oral cavity via breaks in the oral mucosa caused by coarse food, dental
abrasions, or the loss of deciduous teeth. It then migrates up the nerves to the brain
stem, where it stimulates a localized inflammatory response in the form of
microabscesses comprised primarily of neutrophils. It is believed that
L.monocytogenes primarily induces a cell-mediated immune response in the host
and the severity of the resulting lesions may be mediated by the degree of immune
recognition of the organ ism. Microabscesses are most common in the medulla
and lead to destruction of cranial nerve nuclei, notably nerves V through IX. The
cranial nerve defifi cits seen clinically reflfl ect this process. Occasionally,
generalized meningitis can occur in addition to focal encephalitis.
The incubation period in the encephalitic form may be two to three weeks. In the
septicemic form, the incubation period may be as short as one day. The organism is
believed to gain entry through the intestinal mucosa. There is an initial bacteremia
with fever. This may be followed by recov ery, development of a latent carrier state,
or progression to more severe clinical disease. Because themorbidity rate is often low
in outbreaks of septicemic listeriosis, it is presumed that many animals handle
transient bacteremia effectively and are only subclinically infected. When animals do
become ill they may die within forty-eight hours or the illness may last for several
weeks.
Pregnant does abort several days after the initial fever and aborted fetuses also show
evidence of septicemia. Septicemic goats may excrete the organism in feces and milk
during and after clinical illness. Newborn kids exposed to the colostrum or
milk of infected does can show signs of septicemia in the first few days of life.
Seroconversion is marked in goats after septicemic listeriosis but mild in goats after
encephalitic listeriosis . Ocular forms of listeriosis are also reported in cattle and
sheep. Keratoconjunctivitis and iritis appears to result from direct contact of the eye
with Listeria present in silage during the act of feeding (“silage eye”).
Clinical Findings
The encephalitic form is the most common in goats. Though unusual, septicemic and
encephalitic listerio sis have been reported in the same goat herd
The initial signs of the encephalitic form are non specififi c and include
depression,
decreased appetite,
decrease in milk production
transient fever of up to 107.6°F (42°C).
These prodromal signs may be followed by
incoordination
hemiparesis with a tendency for the goat to lean, stumble, or move in one direction
only.
This tendency progresses to obvious torticollis and circling in the same direction.
In advanced cases, the goat may be recumbent with the head pulled tightly into the
flank, unable to straighten the neck voluntarily.
Deficits of the facial nerve are also common and may occur with or without
concurrent hemiparesis and circling. The signs are usually unilateral and
include ear droop, ptosis, flaccid buccal muscles with accumulation of feed in the
buccal pouch, salivation, and a collapsed nostril . Slack jaw, weak
tongue, impaired swallowing, and nystagmus may also be seen. When lesions are
bilateral, some of these deficits may be paradoxically less obvious since the
abnormalities are symmetrical.
Keratitis may be observed as a sequela to abnormal eyelid function. Loss of excessive
saliva through drooling and the inability to swallow can lead to acid-base imbalance,
electrolyte and flfl uid losses, dehydration, and weakness. The course of encephalitic
listeriosis in goats is usually one to four days, which is shorter than that observed in
cattle. The morbidity rate is variable, but the mortality rate can be high.
The septicemic form also begins with
depression,
loss of appetite,
decreased milk production
, and fever up to 107.6°F (42°C).
In these cases, fever may persist and the animal grows progressively weaker over the
next several days.
Neurologic signs rarely develop, but diarrhea, often bloody, is a common finding in
goats.
Goats may die within a few days or remain ill for several weeks. Pregnant does abort
several days after the onset of septicemia.
Clinical Pathology and Necropsy
The hemogram may remain normal, particularly in the encephalitic form of the
disease, or show a neutrophilic leukocytosis. The monocytosis seen in laboratory
animals does not occur in ruminants. Analysis of the cerebrospinal flfl uid may be
helpful. Protein levels and cell counts are typically moderately elevated.
The cells are predominantly monocytes and lymphocytes, with some neutrophils also
present. Bacteria are rarely seen in the CSF, and culture from the CSF is almost
always unrewarding. Historically, serology, while useful for epidemiologic studies,
has not been widely applied for diagno sis of individual cases .
A number of different serodiagnostic techniques have been employed using crude
antigens, but a general limitation for all these techniques was a lack of specificity,
with cross reactions to other Gram-positive organisms being common .
Another practical limitation is that in cases of encephalitic listeriosis, affected animals
do not appear to mount a consistent, detectable humoral immune response, in contrast
to cases of septicemic listeriosis. An increase in indirect hemagglutination titer is
reported after septicemic listeriosis, but not after the encephalitic form .
Sero-agglutination was found to be unsatisfactory as a screening test for accurately
identifying herds with enzootic listeriosis.
ELISA tests have been developed in recent years to detect antibodies against a
specififi c antigen, listerioly sin O, which is an extra-cellular 58 kDa haemolysin,
produced by all the pathogenic strains of L. monocytogenes
Gross post mortem findings are uncommon in the encephalitic form of the disease,
though visible, focal gray discoloration and malacia of the brain stem have
been observed in affected goats .
In the septicemic form, multiple foci of necrosis may be seen in liver, spleen, kidney,
and heart. Multiple, small yellowish spots on the liver of aborted fetuses
are highly suggestive of listeriosis. Placentitis and endometritis may also be observed
in does that abort.
Successful culture is most likely from the liver, spleen, lung, and uterus of septicemic
adults.
Diagnosis
Neurologic diseases that can produce localizing signs consistent with a diagnosis of
listeriosis include the neurologic form of CAE, focal brain abscesses,
cerebrospinal nematodiasis, coenurosis, middle ear infections, bacterial meningitis,
early rabies, and trauma to the facial nerve.
The differential diagnosis for septicemic listeriosis, particularly when diarrhea is
present, includes salmonellosis, yersiniosis, and enterotoxemia. When weakness
predominates and diarrhea is absent, milk fever, and pregnancy toxemia should be
ruled out.
Treatment
Penicillins, tetracyclines, and where permitted, chloramphenicol, are effective
antibiotics.
Adult goats with the septicemic form of disease responded favorably to intramuscular
penicillin administered for three consecutive days at a dose of 2.5 g per day, but
shorter courses of therapy were less effective.
In the encephalitic form, intravenous sodium penicillin at a dose of 40,000 IU/kg
every six hours until improvement is noted, followed by a seven-day course of
intramuscular procaine penicillin at a dose of 20,000 IU/kg twice a day has been
recommended .
Oxytetracycline should be given intravenously at a dose of 10 mg/kg twice a day for
at least three days. These high dosage levels are necessary to promote passage of
antibiotic across the blood brain barrier and development of high-tissue
concentrations in the CNS.
It has been reported that ampicillin or amoxicillin
given in conjunction with gentamicin is the treatment regimen of choice in human
listeriosis cases.
The use of a combination of gentamicin given at a dose of
3 mg/kg bw IV BID and amoxicillin given at a dose of 7 mg/kg bw IM BID was
reported in one retrospective case study in small ruminants .
The outcomes for sheep and goats treated with gentamicin/ampicillin were better
than those treated with either penicillin or oxytetracycline. However, more of
the animals treated with gentamicin/amoxicillin had a favorable prognosis at the onset
of therapy because they were not yet recumbent.
The use of gentamicin in goats is problematic due to prolonged antibiotic residues in
meat and milk.
Dexamethasone given once a day at a dose of 0.1 mg/kg intravenously has also been
used in con junction with antibiotics in the treatment of encephalitic listeriosis with
the rationale that steroids may suppress the infifi ltration of mononuclear cells that
lead to microabscesses in the brain stem.
The nonsteroidal anti-inflfl ammatory drug flunixin meglumine has been used at a
dose of 2.2 mg/kg bw IV SID in the treatment of goats with encephalitic listeriosis but
its contribution to a favorable outcome is not documented.
Supportive therapy in the form of fluid and electrolyte administration, supplemental
feeding, and management of exposure keratitis associated with lid
paralysis may be necessary in severely affected animals. Large amounts of
bicarbonate and fluid may be lost when salivation is prolonged and intense so
fluid therapy should be tailored to address these deficiencies.
Control
In outbreaks of disease, aborting does should be isolated from the herd, and kids
should be raised separately from adults.
Aborted fetuses, placentas, and discharges should be handled wearing gloves and face
masks, and disposed of carefully. Kids should not receive unpasteurized colostrum or
milk from does involved in the outbreak to avoid neonatal septicemia.
Because of the zoonotic potential, no unpasteurized milk should be consumed from
goats in a herd with a history of listeriosis since shedding of the organism in the milk
of latent carrier animals does occur.
A vaccine has been in use in central Europe and Norway to protect sheep from
listeriosis. While case rates in vaccinated and unvaccinated sheep were
similar, the severity of disease was less and the response to treatment better in
vaccinated animals
Goat farmers who produce or market milk, cheese, or other dairy products need to be
aware of the zoonotic potential of these products if contaminated with
L. monocytogenes. There should be no sale of raw milk products from herds where the
Listeria infection status in not known to be negative. Some of the challenges of
keeping cheeses free of listeria contamination in small scale cheese making operations
and approaches to reducing the risk of contamination of soft goat cheeses have been
reported.
Johne’s disease / Paratuberculosis
Paratuberculosis, also known as Johne’s disease, is an economically important
infectious disease of domestic and wild ruminant animals primarily affecting the
digestive tract. Infection leads to gradual debilitation and death through a mechanism
of digestive dysfunction that is not completely understood.
Because the disease has been studied most intensively in cattle, many aspects of the
bovine disease traditionally have been assumed to hold true for goats. However, this
is not the case. Diarrhea, which is the cardinal sign in cattle, is an uncommon clinical
sign in goats. Caprine paratuberculosis is characterized mainly by chronic,progressive
weight loss in adults.
Etiology
Paratuberculosis is caused by the bacterium for merly known as Mycobacterium
paratuberculosis (also previously called M. johnei). Based on advances in the
molecular characterization of the organism that occurred in the 1990s, the organism
has since been reclassified as a subspecies of Mycobacterium avium and
is referred to as M. avium subsp. paratuberculosis (Map).
Analysis of Map strains from different geographic locations and different host species
using various molecular techniques indicates that there are two main groups
of strains. The C, or cattle, strains occur mainly in cattle but are also the predominant
strains in goats. The C strains also occur in deer and, rarely, sheep. The S, or
sheep, strains occur mainly in sheep but also in farmed deer and sometimes goats. The
presence of S strains in goats, though less frequent than the C strains, occurs
mainly when goats are commingled with infected sheep. Restriction endonuclease
analysis and DNA hybridization studies indicate that there may be a
unique Map strain that is isolated from Norwegian goats that falls neither in the C nor
S strain groups, but other studies could not distinguish Norwegian goat strains from
cattle strains.
Map is a small mycobacterium (0.5 × 1 microns) compared with other pathogenic
mycobacteria. It demonstrates typical acid fastness with Ziehl-Neelsen
stain. In tissues, the organism tends to be found in clumps inside macrophages rather
than as individual bacteria.
The bacterium is very resistant to environmental degradation and can persist in
barnyards and in manure spread on pastures for longer than one year. Shade on
pastures prolongs the survival time and thereare indications that Map may be
genetically capable of dormancy, i.e., being able to enter a viable but non cultivable
state and later reverting to a vegetative form . Disinfectants capable of eliminating the
organism from the environment include cresylic compounds diluted 1 : 64 and sodium
ortho phenylphenate diluted 1 : 200.
The C strain of Map is a fastidious, slow-growing organism in vitro. Positive cultures
are rarely identififiable before six weeks and should be held at least twelve
weeks for confirmation of negative cultures. The cultivation of fifi eld isolates of Map
requires supplementation of the culture media with mycobactin, an
iron-chelating substance found in other Mycobacterium spp.
Epidemiology
Paratuberculosis was first definitively described in cattle in 1895 and in goats in
1916 ). It has traditionally been considered a disease of temperate regions with
sporadic occurrence in tropical environments, primarily as a result of importation of
infected livestock from endemic areas. Today, paratuberculosis is generally
considered a disease of worldwide distribution. However, accurate details on the
geographic distribution of caprine paratuberculosis are diffcult to ascertain, because
statistics on goat disease are often reported together with sheep by governments and
international agencies.
Caprine paratuberculosis has been specifically described in numerous countries
representing all continents but Antarctica. These countries include the
Sudan in Africa; India , Nepal, and Korea in Asia; Turkey, and
Israel in the Middle East; Cyprus , France , Greece, Norway), Spain , and Switzerland
in Europe; Canada, the United States , and Mexico in North America; Chilein South
America; and Australia and New Zealand .
This also suggested that paratuberculosis might be a zoonotic disease, potentially
transferable by contact with infected livestock or by consumption of food
products from those animals.
A comprehensive study by the National Research Council published in 2003
concluded that “there remains insuffifi cient evidence to prove or disprove that
Mycobacterium avium subsp. paratuberculosis is a cause
of some or all cases of Crohn’s disease in humans” . However, it was also concluded
that a causal link between paratuberculosis and Crohn’s is plausible and a new
research approach is needed to defifi nitively establish or disprove that relationship.
Nevertheless, there are members of the human medical community who believe quite
strongly that suffifi cient evidence exists to make that link and they call for a strong
and immediate public health response .
Pathogenesis
The primary mode of transmission of paratuberculosis is fecal-oral, with the organism
shed in the manure of infected adults and ingested by susceptible young
stock, particularly when animals are overcrowded and sanitation is poor.
Neonates are considered most susceptible to new infections, particularly when their
own dam is an active fecal shedder and the kid is allowed to remain with the doe.
However, age-related resistance is not absolute, and it is probable that adult animals
may remain at risk for new infection if kept in overcrowded, heavily contaminated
environments.
The role of in utero transmission in the goat remains unknown, although the
organism has been recovered from the uterus and fetal organs of
some experimentally infected goats. If fetal infections were shown to occur naturally
in goats, then currently accepted disease control practices would require modification.
As with cattle and sheep, goats are most likely infected at an early age by ingestion of
the organism. Following ingestion, the organism localizes in the
mucosa of the small intestine and associated lymph nodes. The organism is
transported across the intestinal mucosa by M cells associated with the Peyer’s
patches or by enterocytes
While some exposed individuals may develop resistance to chronic infection, many
infected goats subsequently carry the infection in a dormant state in the
Peyer’s patches of the intestine and the mesenteric lymph nodes for a variable period
into adulthood. At some point, triggered by stress or other ill-defined
factors, some infected animals begin to shed the organism in the feces. They may
begin to show clinical signs concurrently or at a later time.
In general, small ruminants with paratuberculosis tend to have less severe lesions of
granulomatous enteritis than affected cattle. There have been a number
of recent studies of experimental paratuberculosis infection in goats aimed at gaining
a better under standing of the pathogenesis of the disease at a cellular
level and the associated cell mediated and humoral immune responses . A standard
experimental challenge model for caprine paratuberculosis has been proposed.
There is experimental evidence that bacteremia occurs in goats with clinical
paratuberculosis based on isolation of the organism in blood cultures and numer
ous tissues at necropsy, including the udder and uterus . This suggests that offspring
born to clinically infected does have a very high likelihood of infection through the
birth process or when suckling, if not already infected as fetuses.
Because of the prolonged dormancy of infection, the persistence of the organism in
the environment, and the endemic nature of the disease, all goats in a known
infected herd must be considered at risk of infection. Goats in an infected herd may
fall into one of four categories: resistant or non-infected individuals;
infected, non-shedders; subclinically infected, inap parent-shedders; and clinically
affected, apparent shedders. Only individuals in the last group can be identififi ed as
abnormal on physical examination, although a defifi nitive diagnosis can never be
made on physical examination alone. The subclinical infection rate in an infected herd
is likely to be much higher than the rate of clinically apparent cases.
Clinical Findings
Overt clinical disease rarely occurs before one year of age and is most common in
goats two and three years of age. Clinical disease is often triggered by some
episode of stress, such as parturition or recent introduction into a new herd. Affected
individuals begin a course of progressive weight loss, which may extend
from weeks to months and can lead to dramatic emaciation . Appetite may remain
intact initially, but decrease later on, and the animal becomes increasingly lethargic
and depressed. A rough hair coat and flaky skin are common.
Animals in the advanced state of disease may become immunologically anergic.
Advanced cases eventually succumb to debilitation, inanition, exposure, or secondary
infections. Unlike cattle, goats rarely show persistent watery diarrhea, except possibly
in the terminal stages of disease.
As the disease progresses a moderate anemia of chronic infection may develop and
clinical evidence of hypoalbuminemia such as intermandibular edema
may be seen.
Clinical Pathology and Necropsy
Hypergammaglobulinemia and hypocalcemia have also been reported in affected
goats . Definitive diagnosis of paratuberculosis requires bacteriologic or serologic
testing and/or histopathologic examination of tissues.
Infection with paratuberculosis slowly evolves in such a way that no single diagnostic
test can possibly detect all infected animals at any given time in the
course of the disease.
The IFNγ assay was used to monitor the progression of paratuberculosis in a naturally
infected herd of pygmy goats along with fecal culture, ELISA, and
AGID. The IFNγ assay produced both false positive (one of three) and false negative
(three of ten) results in live animals based on the true infection status determined
subsequently by necropsy and culture of Map from tissues . In experimentally
challenged goats, CMI was measured by IFNγ assay, but also by interleukin 2
receptor expression and by lymphocyte proliferation test. CMI responses were
detectable by nine weeks post inoculation and persisted with variation through the two
years of the study, though responses were stronger in the first year.
The tests currently in use for identification of antibody responses in goats are the
complement fixation (CF) test, the agar gel immunodiffusion (AGID) test,
and the ELISA test. In general, when applied to goats, these tests have a sensitivity in
the range of 85% to 100% when applied to clinical cases but only 20% to
50% when applied to subclinical cases (Stehman 2000).
Molecular probes and PCR techniques have also been developed to confirm the
presence of Map-specific genetic material in feces or tissue samples, in an
effort to bypass the need for culture. However, inhibitors in feces and inefficient DNA
release from clinical samples have hampered the general application of
these techniques, particularly with regard to fecal samples.
Most laboratories with the capacity to culture Map still offer the conventional solid
media technique as the main or sole method for detection of Map in feces.
Diagnosis
The differential diagnosis of paratuberculosis in goats is essentially the differential
diagnosis of chronic weight loss.
Treatment
There are no known effective treatments for the elimination of Map infection and
none are specififi cally approved for the treatment of paratuberculosis.
Various antimycobacterial drugs have been used with limited effect in goats.
Attempted unsuccessful therapies have included isoniazid, isoniazid plus rifampin,
isoniazid plus ethambutol, or all three drugs together . However, some treatment
regimens have alleviated clinical signs in individual cases as long as
therapy is continued.y.
One such regimen is daily streptomycin sulfate (0.5 g intramuscularly) plus isoniazid
(25 mg by mouth) plus sodium aminosalicylate (850 mg by mouth) for six months.
A second regimen is daily dihydrostreptomycin (0.5 g intramuscularly) plus rifampin
(300 mg by mouth) and isoniazid (300 mg by mouth) twice daily.
Treatment has little economic application in goats except for the possibility of
keeping known infected, high-quality, breeding does in good health long enough
to achieve embryo transfers or to prolong the life of pet goats.
Control
The second major stimulus for control of paratuberculosis is the much publicized
zoonotic potential of the disease relative to Crohn’s disease in humans, which
has caused concern among public health officials, regulatory veterinary agencies, and
livestock producers about public perception of milk and meat as whole
some foods. As a result, numerous countries have initiated or expanded their
paratuberculosis control programs in recent years. Most programs focus on
cattle, but some may include sheep and goats.
Improved management and sanitation help reduce the incidence of new infections in
young stock. In intensive dairy operations, kidding pens should be
cleaned or rebedded between kiddings.
Kids should be taken immediately from the does and raised in separate facilities on
heat-treated colostrum and pasteurized milk or milk replacer.
Kids should not commingle again with adult animals until they themselves have
kidded. Adult animals should not be over crowded, manure should be removed
frequently from the pens, and feed and water should be provided in a manner that
does not allow for fecal contamination.
If manure is spread on pastures, goats should not be allowed to graze those pastures
for at least one year. The use of pastures in general may be problematic as it has
become clear that wildlife reservoirs exist for Map which may contaminate pastures,
even when paratuberculous livestock have been kept off those pastures.
Toxoplasmosis
Etiology
The protozoan Toxoplasma gondii is a very important cause of abortion,
mummifification, stillbirth, and birth of weak young in goats as it is in sheep.
Epidemiology and Pathogenesis.
Cats serve as the defifi nitive host for this parasite they become infected by
consuming uncooked meat scraps, placentas, and small rodents . Recently infected
cats then shed oocysts in their feces.
Oocyst shedding typically lasts from three to nineteen days, but oocysts may persist in
moist and shaded soil for as long as eighteen months.
Goats become infected by eating grass, hay, or grain contaminated by cat feces. After
initial invasion of the goat’s small intestine and associated lymph nodes, the
Toxoplasma organisms spread via the bloodstream to other tissues, including muscle,
brain, and liver. Here the parasite may remain encysted for months or even the life of
the goat. If the goat is pregnant at the time of initial infection, Toxoplasma commonly
invade the placenta and fetus approximately two weeks after initial infection of the
doe. Fetuses infected in the first half of pregnancy are more apt to die than fetuses
infected in the second half. Sometimes abortion is repeated in the next gestation , but
previously infected goats are usually resistant to abortion or other clinical signs when
challenged with T. gondii
Diagnosis
Fetal serology is a very specififi c test for abortive toxoplasmosis . The ovine fetus
(and presumably the caprine fetus) begins to develop immunocompetency at sixty to
seventy days of gestation.
First immunoglobin M is produced, but by ninety days immunoglobin G is being
synthesized. Thus, if the fetus is infected after midgestation, it will usually produce
antibody detectable by a variety of immunologic tests. A modified direct agglutination
test (MAT) is considered to be very sensitive and can be used on any species,
including goats, because species-specififi c conjugates are not used .
Heart blood or thoracic flfl uid can be harvested from the fetus. Fetal autolysis does
not always preclude identification of toxoplasma antibodies. Absence of
antibodies in the fetus does not rule out toxoplasmosis because the fetus may have
been infected too young or antibodies may have decomposed. Screening at
both high and low dilutions is recommended to avoid false negatives when the
antibody concentration is so high as to occupy all binding sites.
Serologic testing of the doe at time of abortion is useful. Absence of antibodies is
considered to be conclusive evidence that toxoplasmosis was not the cause of
abortion .
If the diagnosis is to be made by histology, it is very important that placenta be
submitted. Small yellowish-white foci of mineralization confined to
cotyledons are apparent grossly if abortion is delayed until forty-five days or more
after infection. Washing the cotyledons thoroughly in isotonic saline solution
makes deeper foci easier to visualize. Another useful technique is to compress the
cotyledon with a glass microscope slide; the mineralized foci resist squashing.
Microscopic foci of necrosis can be identififi ed after thirty days; tachyzooites are
sparse in these lesions and diffifi cult to locate (Dubey 1988). Nonsuppurative
encephalomyelitis is found more consistently than myocarditis, but even under ideal
experimental conditions it is diffifi cult to fifi nd T. gondii organisms in tissue
sections.
Prevention
To prevent exposure of susceptible goats to the oocysts in cat feces during the
period of danger, which is pregnancy.
In particular, grain should be stored in covered containers and the mangers kept clean.
Contamination of the hay supply (by cats living in the hay barn) has been implicated
in several outbreaks . If possible, feed the hay off the top of the stack to the
nonpregnant does and young stock.
A vasectomized tomcat might be helpful in keeping stray cats off the farm, but this
technique has not been evaluated. Raw meat should not be fed to cats
A live toxoplasmosis vaccine available for sheep in the UK might be effective in
goats . Repeated administration of killed vaccine or vaccination with a related but
nonpathogenic organism (Hammondia) might be effective in preventing abortion
but would certainly be expensive; such vaccines are unlikely to be
marketed in the near future.
When abortions caused by toxoplasmosis are diagnosed, emphasis should be put on
properly disposing fetuses and placentas, wearing protective gloves when
handling these items, and properly pasteurizing milk and cooking meat. Pregnant
women should be especially careful.
Causes of outbreak of an abortion in goats during the scarcity of green fodder

Weitere ähnliche Inhalte

Was ist angesagt?

Minerals Deficiencies in Poultry Causes ,Effect & Treatment.
Minerals  Deficiencies in  Poultry Causes ,Effect & Treatment.Minerals  Deficiencies in  Poultry Causes ,Effect & Treatment.
Minerals Deficiencies in Poultry Causes ,Effect & Treatment.
Growel Agrovet Private Limited
 

Was ist angesagt? (20)

Management of high yielders during transition period
Management of high yielders during transition periodManagement of high yielders during transition period
Management of high yielders during transition period
 
Minerals Deficiencies in Poultry Causes ,Effect & Treatment.
Minerals  Deficiencies in  Poultry Causes ,Effect & Treatment.Minerals  Deficiencies in  Poultry Causes ,Effect & Treatment.
Minerals Deficiencies in Poultry Causes ,Effect & Treatment.
 
Nutritional Factors Affecting Hatchability
Nutritional Factors Affecting HatchabilityNutritional Factors Affecting Hatchability
Nutritional Factors Affecting Hatchability
 
Cattle heat detection and heat signs
Cattle heat detection and heat signsCattle heat detection and heat signs
Cattle heat detection and heat signs
 
Metabolic disorders in dairy animals
Metabolic disorders in dairy animalsMetabolic disorders in dairy animals
Metabolic disorders in dairy animals
 
Repeat breeding by Dr Adnan ahmad
Repeat breeding by Dr Adnan ahmadRepeat breeding by Dr Adnan ahmad
Repeat breeding by Dr Adnan ahmad
 
Induction of parturition & elective termination of pregnancy
Induction of parturition & elective termination of pregnancyInduction of parturition & elective termination of pregnancy
Induction of parturition & elective termination of pregnancy
 
milk fever
milk fevermilk fever
milk fever
 
Downer's cow syndrome
Downer's cow syndromeDowner's cow syndrome
Downer's cow syndrome
 
Calf diarrhoea
Calf diarrhoeaCalf diarrhoea
Calf diarrhoea
 
Etiopathogenesis, therapy, prevention and control of milk fever in dairy cattle
Etiopathogenesis, therapy, prevention and control of milk fever in dairy cattleEtiopathogenesis, therapy, prevention and control of milk fever in dairy cattle
Etiopathogenesis, therapy, prevention and control of milk fever in dairy cattle
 
Milk fever
Milk feverMilk fever
Milk fever
 
Nutrition for commercial broiler and layer production
Nutrition for commercial broiler and layer productionNutrition for commercial broiler and layer production
Nutrition for commercial broiler and layer production
 
Bovine ketosis
Bovine ketosisBovine ketosis
Bovine ketosis
 
Importance of Vitamins and Minerals for Dairy Cattle , Published in Dairy Pla...
Importance of Vitamins and Minerals for Dairy Cattle , Published in Dairy Pla...Importance of Vitamins and Minerals for Dairy Cattle , Published in Dairy Pla...
Importance of Vitamins and Minerals for Dairy Cattle , Published in Dairy Pla...
 
Milk replacer and calf starter
Milk replacer and calf starterMilk replacer and calf starter
Milk replacer and calf starter
 
Lecture 2: Estrus Detection in Cattle
Lecture 2: Estrus Detection in Cattle Lecture 2: Estrus Detection in Cattle
Lecture 2: Estrus Detection in Cattle
 
Non Protein Nutrogen Utilization in Ruminant
Non Protein Nutrogen Utilization in RuminantNon Protein Nutrogen Utilization in Ruminant
Non Protein Nutrogen Utilization in Ruminant
 
Lecture 5 follicular dynamics in domestic animals
Lecture 5 follicular dynamics in domestic animalsLecture 5 follicular dynamics in domestic animals
Lecture 5 follicular dynamics in domestic animals
 
Lecture 9 Ovarian cysts in domestic animals
Lecture 9 Ovarian cysts in domestic animalsLecture 9 Ovarian cysts in domestic animals
Lecture 9 Ovarian cysts in domestic animals
 

Ähnlich wie Causes of outbreak of an abortion in goats during the scarcity of green fodder

6. different kinds of swine diseases
6. different kinds of swine diseases6. different kinds of swine diseases
6. different kinds of swine diseases
Rudy Flores
 
Salmonella infections
Salmonella infectionsSalmonella infections
Salmonella infections
Jasmine John
 

Ähnlich wie Causes of outbreak of an abortion in goats during the scarcity of green fodder (20)

Typhoid Fever
Typhoid FeverTyphoid Fever
Typhoid Fever
 
6. different kinds of swine diseases
6. different kinds of swine diseases6. different kinds of swine diseases
6. different kinds of swine diseases
 
Red water disease Allah Dad Khan
Red water disease Allah Dad KhanRed water disease Allah Dad Khan
Red water disease Allah Dad Khan
 
Listeriosis.ppt
Listeriosis.pptListeriosis.ppt
Listeriosis.ppt
 
Lecture Day 3 And 4
Lecture Day 3 And 4Lecture Day 3 And 4
Lecture Day 3 And 4
 
Livestock diseases
Livestock diseasesLivestock diseases
Livestock diseases
 
Livestock diseases
Livestock diseasesLivestock diseases
Livestock diseases
 
Bacterial food born diseases
Bacterial food born diseasesBacterial food born diseases
Bacterial food born diseases
 
Toxoplasmosis and Q-Fever
Toxoplasmosis and Q-FeverToxoplasmosis and Q-Fever
Toxoplasmosis and Q-Fever
 
Salmonella infections
Salmonella infectionsSalmonella infections
Salmonella infections
 
Food poisoning.pptx
Food poisoning.pptxFood poisoning.pptx
Food poisoning.pptx
 
Food borne Diseases unit.pptx
Food borne Diseases unit.pptxFood borne Diseases unit.pptx
Food borne Diseases unit.pptx
 
Typhoid and paratyphoid
Typhoid and paratyphoidTyphoid and paratyphoid
Typhoid and paratyphoid
 
Ofooni1_09_Cholera_Campylobacter.ppt
Ofooni1_09_Cholera_Campylobacter.pptOfooni1_09_Cholera_Campylobacter.ppt
Ofooni1_09_Cholera_Campylobacter.ppt
 
disease management in livestock and its prevention
disease management in livestock and its preventiondisease management in livestock and its prevention
disease management in livestock and its prevention
 
Food Poisoning.ppt
Food Poisoning.pptFood Poisoning.ppt
Food Poisoning.ppt
 
Ppt food poisoning
Ppt food poisoningPpt food poisoning
Ppt food poisoning
 
Epidemiology of kfd, brucellosis and leptospirosis
Epidemiology of kfd, brucellosis and leptospirosisEpidemiology of kfd, brucellosis and leptospirosis
Epidemiology of kfd, brucellosis and leptospirosis
 
Enumeration of salmonella and shigella
Enumeration of salmonella and shigellaEnumeration of salmonella and shigella
Enumeration of salmonella and shigella
 
Osun
OsunOsun
Osun
 

Mehr von DR AMEER HAMZA

Mehr von DR AMEER HAMZA (20)

Clinical examination of animals
Clinical examination of animalsClinical examination of animals
Clinical examination of animals
 
Displacement of the abomasum in cattle
Displacement of the abomasum in cattleDisplacement of the abomasum in cattle
Displacement of the abomasum in cattle
 
Postpartum Hypocalcemia , Periparturient Hypocalcemia , Puerperal Tetany , E...
 Postpartum Hypocalcemia , Periparturient Hypocalcemia , Puerperal Tetany , E... Postpartum Hypocalcemia , Periparturient Hypocalcemia , Puerperal Tetany , E...
Postpartum Hypocalcemia , Periparturient Hypocalcemia , Puerperal Tetany , E...
 
Diseases responsible for blindness in cattle
Diseases responsible for blindness in cattle Diseases responsible for blindness in cattle
Diseases responsible for blindness in cattle
 
Lameness diagnoses in equines
Lameness  diagnoses in equinesLameness  diagnoses in equines
Lameness diagnoses in equines
 
Superficial digital tendon in equines
Superficial  digital tendon in equinesSuperficial  digital tendon in equines
Superficial digital tendon in equines
 
Laminitis IN EQUINES
Laminitis IN EQUINESLaminitis IN EQUINES
Laminitis IN EQUINES
 
SHOEING PROCEDURE IN HORSES
SHOEING PROCEDURE IN HORSESSHOEING PROCEDURE IN HORSES
SHOEING PROCEDURE IN HORSES
 
Quittor IN EQUINES
Quittor IN EQUINESQuittor IN EQUINES
Quittor IN EQUINES
 
EQUINE SOUNDNESS CERTIFICATE
EQUINE SOUNDNESS CERTIFICATEEQUINE SOUNDNESS CERTIFICATE
EQUINE SOUNDNESS CERTIFICATE
 
LOSS OF HORSESHOE
LOSS OF HORSESHOELOSS OF HORSESHOE
LOSS OF HORSESHOE
 
SHOCK ABSORBER IN HOOF
SHOCK ABSORBER IN HOOFSHOCK ABSORBER IN HOOF
SHOCK ABSORBER IN HOOF
 
DIAGNOSIS OF LAMENESS IN HORSES
DIAGNOSIS OF LAMENESS  IN HORSESDIAGNOSIS OF LAMENESS  IN HORSES
DIAGNOSIS OF LAMENESS IN HORSES
 
TENDONS & LIGAMENTS OF EQUINE FOOT
TENDONS & LIGAMENTS  OF EQUINE FOOTTENDONS & LIGAMENTS  OF EQUINE FOOT
TENDONS & LIGAMENTS OF EQUINE FOOT
 
SURGICAL MANAGEMENT OF TAIL IN VETERINARY
SURGICAL MANAGEMENT OF TAIL IN VETERINARYSURGICAL MANAGEMENT OF TAIL IN VETERINARY
SURGICAL MANAGEMENT OF TAIL IN VETERINARY
 
FIRING AND COUNTER IRRITANTS USED IN VETERINARY
FIRING AND COUNTER IRRITANTS USED  IN VETERINARYFIRING AND COUNTER IRRITANTS USED  IN VETERINARY
FIRING AND COUNTER IRRITANTS USED IN VETERINARY
 
INDUCTION AND MAINTANANCE OF ANESTHESIA IN FIELD CONDITION IN ANIMALS
INDUCTION AND MAINTANANCE OF ANESTHESIA IN FIELD CONDITION IN ANIMALSINDUCTION AND MAINTANANCE OF ANESTHESIA IN FIELD CONDITION IN ANIMALS
INDUCTION AND MAINTANANCE OF ANESTHESIA IN FIELD CONDITION IN ANIMALS
 
HEAD REGION NERVE BLOCK IN CAT & DOGS
HEAD REGION NERVE BLOCK  IN CAT & DOGS HEAD REGION NERVE BLOCK  IN CAT & DOGS
HEAD REGION NERVE BLOCK IN CAT & DOGS
 
The anesthesia of birds and lab animals
The anesthesia of birds and lab animalsThe anesthesia of birds and lab animals
The anesthesia of birds and lab animals
 
ROLE OF ANTI-INFLAMMATORY DRUGS IN SURGERY
ROLE OF ANTI-INFLAMMATORY DRUGS  IN  SURGERYROLE OF ANTI-INFLAMMATORY DRUGS  IN  SURGERY
ROLE OF ANTI-INFLAMMATORY DRUGS IN SURGERY
 

Kürzlich hochgeladen

Call Girls Bhubaneswar Just Call 9907093804 Top Class Call Girl Service Avail...
Call Girls Bhubaneswar Just Call 9907093804 Top Class Call Girl Service Avail...Call Girls Bhubaneswar Just Call 9907093804 Top Class Call Girl Service Avail...
Call Girls Bhubaneswar Just Call 9907093804 Top Class Call Girl Service Avail...
Dipal Arora
 
Call Girls in Gagan Vihar (delhi) call me [🔝 9953056974 🔝] escort service 24X7
Call Girls in Gagan Vihar (delhi) call me [🔝  9953056974 🔝] escort service 24X7Call Girls in Gagan Vihar (delhi) call me [🔝  9953056974 🔝] escort service 24X7
Call Girls in Gagan Vihar (delhi) call me [🔝 9953056974 🔝] escort service 24X7
9953056974 Low Rate Call Girls In Saket, Delhi NCR
 
College Call Girls in Haridwar 9667172968 Short 4000 Night 10000 Best call gi...
College Call Girls in Haridwar 9667172968 Short 4000 Night 10000 Best call gi...College Call Girls in Haridwar 9667172968 Short 4000 Night 10000 Best call gi...
College Call Girls in Haridwar 9667172968 Short 4000 Night 10000 Best call gi...
perfect solution
 

Kürzlich hochgeladen (20)

Call Girls Gwalior Just Call 8617370543 Top Class Call Girl Service Available
Call Girls Gwalior Just Call 8617370543 Top Class Call Girl Service AvailableCall Girls Gwalior Just Call 8617370543 Top Class Call Girl Service Available
Call Girls Gwalior Just Call 8617370543 Top Class Call Girl Service Available
 
Call Girls in Delhi Triveni Complex Escort Service(🔝))/WhatsApp 97111⇛47426
Call Girls in Delhi Triveni Complex Escort Service(🔝))/WhatsApp 97111⇛47426Call Girls in Delhi Triveni Complex Escort Service(🔝))/WhatsApp 97111⇛47426
Call Girls in Delhi Triveni Complex Escort Service(🔝))/WhatsApp 97111⇛47426
 
Call Girls Guntur Just Call 8250077686 Top Class Call Girl Service Available
Call Girls Guntur  Just Call 8250077686 Top Class Call Girl Service AvailableCall Girls Guntur  Just Call 8250077686 Top Class Call Girl Service Available
Call Girls Guntur Just Call 8250077686 Top Class Call Girl Service Available
 
Call Girls Bareilly Just Call 8250077686 Top Class Call Girl Service Available
Call Girls Bareilly Just Call 8250077686 Top Class Call Girl Service AvailableCall Girls Bareilly Just Call 8250077686 Top Class Call Girl Service Available
Call Girls Bareilly Just Call 8250077686 Top Class Call Girl Service Available
 
Call Girls Varanasi Just Call 8250077686 Top Class Call Girl Service Available
Call Girls Varanasi Just Call 8250077686 Top Class Call Girl Service AvailableCall Girls Varanasi Just Call 8250077686 Top Class Call Girl Service Available
Call Girls Varanasi Just Call 8250077686 Top Class Call Girl Service Available
 
Premium Call Girls In Jaipur {8445551418} ❤️VVIP SEEMA Call Girl in Jaipur Ra...
Premium Call Girls In Jaipur {8445551418} ❤️VVIP SEEMA Call Girl in Jaipur Ra...Premium Call Girls In Jaipur {8445551418} ❤️VVIP SEEMA Call Girl in Jaipur Ra...
Premium Call Girls In Jaipur {8445551418} ❤️VVIP SEEMA Call Girl in Jaipur Ra...
 
Call Girls Bhubaneswar Just Call 9907093804 Top Class Call Girl Service Avail...
Call Girls Bhubaneswar Just Call 9907093804 Top Class Call Girl Service Avail...Call Girls Bhubaneswar Just Call 9907093804 Top Class Call Girl Service Avail...
Call Girls Bhubaneswar Just Call 9907093804 Top Class Call Girl Service Avail...
 
Call Girls in Gagan Vihar (delhi) call me [🔝 9953056974 🔝] escort service 24X7
Call Girls in Gagan Vihar (delhi) call me [🔝  9953056974 🔝] escort service 24X7Call Girls in Gagan Vihar (delhi) call me [🔝  9953056974 🔝] escort service 24X7
Call Girls in Gagan Vihar (delhi) call me [🔝 9953056974 🔝] escort service 24X7
 
Call Girls Ludhiana Just Call 9907093804 Top Class Call Girl Service Available
Call Girls Ludhiana Just Call 9907093804 Top Class Call Girl Service AvailableCall Girls Ludhiana Just Call 9907093804 Top Class Call Girl Service Available
Call Girls Ludhiana Just Call 9907093804 Top Class Call Girl Service Available
 
College Call Girls in Haridwar 9667172968 Short 4000 Night 10000 Best call gi...
College Call Girls in Haridwar 9667172968 Short 4000 Night 10000 Best call gi...College Call Girls in Haridwar 9667172968 Short 4000 Night 10000 Best call gi...
College Call Girls in Haridwar 9667172968 Short 4000 Night 10000 Best call gi...
 
Top Quality Call Girl Service Kalyanpur 6378878445 Available Call Girls Any Time
Top Quality Call Girl Service Kalyanpur 6378878445 Available Call Girls Any TimeTop Quality Call Girl Service Kalyanpur 6378878445 Available Call Girls Any Time
Top Quality Call Girl Service Kalyanpur 6378878445 Available Call Girls Any Time
 
(Low Rate RASHMI ) Rate Of Call Girls Jaipur ❣ 8445551418 ❣ Elite Models & Ce...
(Low Rate RASHMI ) Rate Of Call Girls Jaipur ❣ 8445551418 ❣ Elite Models & Ce...(Low Rate RASHMI ) Rate Of Call Girls Jaipur ❣ 8445551418 ❣ Elite Models & Ce...
(Low Rate RASHMI ) Rate Of Call Girls Jaipur ❣ 8445551418 ❣ Elite Models & Ce...
 
♛VVIP Hyderabad Call Girls Chintalkunta🖕7001035870🖕Riya Kappor Top Call Girl ...
♛VVIP Hyderabad Call Girls Chintalkunta🖕7001035870🖕Riya Kappor Top Call Girl ...♛VVIP Hyderabad Call Girls Chintalkunta🖕7001035870🖕Riya Kappor Top Call Girl ...
♛VVIP Hyderabad Call Girls Chintalkunta🖕7001035870🖕Riya Kappor Top Call Girl ...
 
Night 7k to 12k Navi Mumbai Call Girl Photo 👉 BOOK NOW 9833363713 👈 ♀️ night ...
Night 7k to 12k Navi Mumbai Call Girl Photo 👉 BOOK NOW 9833363713 👈 ♀️ night ...Night 7k to 12k Navi Mumbai Call Girl Photo 👉 BOOK NOW 9833363713 👈 ♀️ night ...
Night 7k to 12k Navi Mumbai Call Girl Photo 👉 BOOK NOW 9833363713 👈 ♀️ night ...
 
Call Girls Jabalpur Just Call 8250077686 Top Class Call Girl Service Available
Call Girls Jabalpur Just Call 8250077686 Top Class Call Girl Service AvailableCall Girls Jabalpur Just Call 8250077686 Top Class Call Girl Service Available
Call Girls Jabalpur Just Call 8250077686 Top Class Call Girl Service Available
 
Top Rated Bangalore Call Girls Richmond Circle ⟟ 9332606886 ⟟ Call Me For Ge...
Top Rated Bangalore Call Girls Richmond Circle ⟟  9332606886 ⟟ Call Me For Ge...Top Rated Bangalore Call Girls Richmond Circle ⟟  9332606886 ⟟ Call Me For Ge...
Top Rated Bangalore Call Girls Richmond Circle ⟟ 9332606886 ⟟ Call Me For Ge...
 
The Most Attractive Hyderabad Call Girls Kothapet 𖠋 9332606886 𖠋 Will You Mis...
The Most Attractive Hyderabad Call Girls Kothapet 𖠋 9332606886 𖠋 Will You Mis...The Most Attractive Hyderabad Call Girls Kothapet 𖠋 9332606886 𖠋 Will You Mis...
The Most Attractive Hyderabad Call Girls Kothapet 𖠋 9332606886 𖠋 Will You Mis...
 
Call Girls Siliguri Just Call 8250077686 Top Class Call Girl Service Available
Call Girls Siliguri Just Call 8250077686 Top Class Call Girl Service AvailableCall Girls Siliguri Just Call 8250077686 Top Class Call Girl Service Available
Call Girls Siliguri Just Call 8250077686 Top Class Call Girl Service Available
 
Best Rate (Hyderabad) Call Girls Jahanuma ⟟ 8250192130 ⟟ High Class Call Girl...
Best Rate (Hyderabad) Call Girls Jahanuma ⟟ 8250192130 ⟟ High Class Call Girl...Best Rate (Hyderabad) Call Girls Jahanuma ⟟ 8250192130 ⟟ High Class Call Girl...
Best Rate (Hyderabad) Call Girls Jahanuma ⟟ 8250192130 ⟟ High Class Call Girl...
 
Best Rate (Patna ) Call Girls Patna ⟟ 8617370543 ⟟ High Class Call Girl In 5 ...
Best Rate (Patna ) Call Girls Patna ⟟ 8617370543 ⟟ High Class Call Girl In 5 ...Best Rate (Patna ) Call Girls Patna ⟟ 8617370543 ⟟ High Class Call Girl In 5 ...
Best Rate (Patna ) Call Girls Patna ⟟ 8617370543 ⟟ High Class Call Girl In 5 ...
 

Causes of outbreak of an abortion in goats during the scarcity of green fodder

  • 1. NAME :AMEER HAMZA CLASS# 01 DVM FINAL YEAR (SEMESTER:09) SEC:A SESSION:2016-2020 MEDICINE CLINIC 3 ASSIGNMENT SUBMITTED TO : DR HAQ AMAN ULLAH Assignment No. 1 In the field you face an outbreakof abortionin goats during scarcityof green fodder. What causes of abortionyou will consider, explain etiology, pathogenesis,clinicalfindings, treatment and preventionof each cause.
  • 2. CAUSES OF ABORTION IN GOATS DURING SCARCITY OF GREEN FODDER : 1. Enterotoxemia 2. Severe stress 3. Exposure to toxic plants, such as broomweed broomweed, turpentine weed 4. Angora abortion syndrome 5. Micronutrient deficiencies copper, selenium, magnesium, vitamin A,E . 6. Bluetongue virus 7. Listeriosis (Circling disease) 8. Johne’s disease 9. Toxoplasmosis ENTEROTOXEMIA ETIOLOGY: It’s a frequently severe disease of goats of all ages. It is caused by two strains of bacteria called Clostridium perfringens – the strains are termed types C and D(Pulpy kidney disease, Overeating disease). These bacteria are normally found in low numbers in the gastrointestinal tract of goats.
  • 3. These organisms are normally “laying low” in the small and large intestine – that is, they are present in relatively low numbers and appear to be in a relatively quiescent state in the normal, healthy animal. Most commonly, the change that triggers disease is an increase in the amount of grain, protein supplement, milk or milk replacer (for kids), or grass that the sheep or goat is ingesting. Collectively, these feeds are rich in starch, sugar, or protein. PATHOGENESIS : Sudden ingestion of readily fermentable, carbohydrate-rich feeds permits more undigested starch to pass through the rumen to the abomasum and intestine where it serves as a nutrient substrate for rapid proliferation of the organism. Excess carbohydrate intake may also predispose to reduced motility. This proliferation of C. perfringens type D in conjunction with reduced peristalsis enhances the concentration and pathogenic potential of the epsilon toxin produced by the organism. Epsilon toxin, after being converted from the prototoxin by intestinal trypsin, increases vascular permeability in the gut, thereby facilitating its own absorption into the blood stream. A generalized toxemia ensues. The toxin is necrotizing, and specififi cally neurotoxic. Death is attributable to damage of vital neurons, generalized toxemia, and shock. Kids showed more prominent diarrhea and abdominal discomfort with fewer neurologic signs preceding death. At necropsy, intestinal lesions in kids, a severe necrotizing colitis was seen grossly and confifirmed histologically. CLINICAL SIGNS : Three distinct clinical forms of enterotoxemia are recognized in goats, namely peracute, acute, and chronic. Peracute form:
  • 4. occurs more frequently in young goats than adults. The clinical course is usually less than twenty-four hours and may go unnoticed. Finding one or more dead animals is often indication of peracute enterotoxemia in a herd. Clinical signs include a sudden loss of appetite, profound depression, marked abdominal discomfort manifested by arching of the back and kicking at the belly, loud and painful screaming, and profuse watery diarrhea containing blood and shreds of mucus. Fevers of 105°F (40.5°C) are recorded. Affected goats quickly become weak and recumbent. They may show paddling or convulsions, but frequently they just lapse into a coma without excitatory signs. Death ensues within hours. Recoveries are rare, even with treatment. Acute form: similar clinical signs are seen but with less severity. Abdominal pain and screaming may be absent or reduced. Feces may first turn pasty or soft, but then become watery. The clinical course lasts from three to four days. Severe dehydration and acidosis, caused by the profuse diarrhea, become complicating factors in these cases. Spontaneous recoveries may occur, but most animals die if not treated. The acute form more often affects mature goats. It can occur in herds with a solid history of vaccination against C. perfringens type D, so enterotoxemia should not be ruled out on the basis of previous vaccination. Chronic form: intermittent, recurring bouts of illness are observed over several weeks. Mature animals are usually affected. These goats are dull and listless with reduced appetite and milk production if lactating. There is progressive weight loss with intermittent episodes of pasty or loose feces. Abortion occur in pregnant animals. Treatment The prognosis for recovery is guarded in caprine enterotoxemia, even with treatment. 1. Antibiotics like penicillin ,amoxicillin,ampicillin etc for 5 days as per recommended dose . 2. Intravenous fluid therapy providing mixed electrolyte solutions with bicarbonate is indicated in peracute and acute cases to counter shock, dehydration, and acidosis. 3.Nonsteroidal anti-inflammatory drugs such as flunixin meglumine (1.1-2.2 mg/kg I/m every twelve hours) may be helpful in stabilizing animals in toxemic shock and will alleviate pain. 4.Commercially available type C and D antitoxins should be administered parenterally, preferably intravenously, in severe cases. Though recommended prophylactic doses are usually around 5 ml, therapeutic doses up to 100 ml have been administered. Because the antitoxin products are relatively expensive, a minimum effective dose is desirable. In the chronic form of the disease, 2 doses of 20 ml of antitoxin given four days apart was reported to be a reasonably effective treatment
  • 5. Prevention Prevention of enterotoxemia is far more likely to be successful than trying to treat the disease. 1. Vaccination For sheep and goats, there are multiple vaccines available that induce immunity to the toxins generated by Clostridium perfringens types C and D. 2. Feeding Strategies Smart feeding strategies i.e always make feed changes slowly STRESS A stress or stressor is a force external to a system that acts to displace the system. A stress condition can be quantified and applied equally across animals. ETIOLOGY: The environmental comfort zone for goats is between 0-30C. Above 30C goats may begin to experience mild heat stress, especially when humidity cranks up the heat index. As heat and humidity climb, goats can have serious problems with thermal stress. High temperatures affect body function in many ways. The hypothalamus, lying at the base of the brain, is in charge of balancing the body’s heat loss and gain by regulating respiration, skin temperature, sweating and muscle tone.When the weather warms up, animals eat less in an involuntary effort to reduce body heat, feed and water consumption go down. Animals may reduce water intake however they need water to help keep them cooler. Weather and other factors can combine to get any goat, but some are more susceptible to overheating. Overweight goats cannot exchange heat efficiently. Selenium deficiency may exacerbate heat stress due to marginal muscle tone. Unventilated confinement, such as being locked in a poorly ventilated barn, crate or vehicle, can be a serious threat to an animal’s life in a very short time. PATHOGENESIS:
  • 6. Nutritional,heat stress negatively affects the ability of an animal to become pregnant through many mechanisms affecting fertilization, follicular development and early embryonic development. Heat stress during pregnancy slows down the growth of the foetus, which was attributed to the decreased uterine blood supply, which hampers supply of nutrients and hormones to the conceptus . Slow growing embryos fail to signal pregnancy to the maternal organism in due time. Therefore, the endometrial prostaglandin F2alpha (PGF2α) secretion tends to increase during heat stress and trigger luteolysis, thereby threatening the maintenance of pregnancy . The placental weight and hormonal secretions are reduced and the vascular resistance is increased during heat stress, which further affects the reduction in perfusion of nutrients to the foetus . CLINICAL SIGNS : 1. Panting rapidly. Animals don’t sweat, instead they pant to relieve themselves of heat. An open mouth and foam around mouth means they are extremely hot. 2. High body temp, anything over 103.2 in hot weather means they are too hot and can’t cool of. 3. Droopy ears, sunken eyes, lack of interest in food/water/herd.
  • 7. 4. Stretched out on ground and does not make an attempt to get up or moved when approached. TREATMENT:  FLUID THERAPY I.e NORMAL SALINE + VIT B COMPLEX  MINERALS PROVISION I.E ORS etc  To sustain pregnancy progesterone drugs like GLOBINAN,PROGESTERONE,PROLUTON DEPOT can be used. PREVENTION:  Ensure there is unlimited access to clean, cool water for all animals.  Reduce stocking densities.  shade that encourages air flow  Temporarily reduce or cease feeding of concentrate and consider a higher roughage proportion in ration until other emergency measures are implemented. Exposure to toxic plants Snakeweed, broomweed, turpentine weed. ETIOLOGY: Botanic Name Gutierrezia sarothrae Plant Family Asteraceae Habitat Dry plains and foothills at altitudes from 4000-10000 feet. Animals Affected Horses, cattle, sheep, goats Toxic Principle Saponins are believed to be the toxic component of snakeweed. Both the green and dried plant are toxic,
  • 8. although there appears to be considerable variability in toxicity. Description Perennial which is shrubby or woody only at the base, attaining 18-24 inches in height. The stems are branching, the leaves are linear and glabrous. The heads are many, usually in clusters at the ends of branches. A given head will have no more than 3-8 ray flowers and 3-8 disc flowers. The flowers are yellow with the disc flowers usually perfect. The corollas are 5-lobed and the pappus is composed of several to many oblong scales. Often forms dense stands especially in overgrazed rangeland. CLINICAL SIGNS: Gastrointestinal Diarrhea followed by constipation. Musculoskeletal Loss of weight Congenital Defects Abortions. Calves may be born alive and weak, and may die after a few days. Reproductive System Abortions, retained fetal membranes. May also decrease male fertility. Hepatic System Liver necrosis and degeneration may be seen at postmortem examination. Diagnosis Abortions, liver necrosis, access to snakeweed. Treatment There is no specific treatment. Affected animals should be removed from the snakeweed and giving supportive care.( MULTIVITAMINS ,ANTIHISTAMINES ,FLUID THERAPY) Enzootic Abortion(Chlamydiosis) /Angora Abortion Syndrome
  • 9. ETIOLOGY: is a contagious disease in Angora goats caused by Chlamydia psittaci. Outbreaks are usually seen a year or two after new goats were introduced onto a farm. The abortions are more likely to be seen under intensive rather than a veld grazing environment. Flocks infected for the first time may have up to 70% abortions. These can occur as early as 3 months into pregnancy where the foetus may be resorbed instead of being expelled. Small weak kids may be born that die soon after birth. Kids can survive and carry the disease. The ewe at no time becomes ill although Chlamydia can be the cause of infectious opthalmia. It is unlikely that the same ewe will abort again due to Chlamydia and hence the reason why that after the initial outbreak abortions are then often seen in maiden ewes on a contaminated farm. PATHOGENESIS: The organisms localise in the epithelial cells of the mucosa of the small intestine and then spread via the blood and lymph vessels. When the organism invades the placenta the damage can result in embryonal death, resorption, mummification or abortion. Less extensive damage results in premature or weak full-term kids. Chlamydia is usually spread during kidding and is transmitted when goat ewes are grazing plant material that has been contaminated by afterbirth or uterine fluid. Venereal transmission is also possible when rams with infected accessory sex glands transmit C. psittaci in their semen. The infection can remain dormant (carrier state) in the ewe kid until she aborts for the first time. The disease may also flare up when carrier animals become stressed. Clinical signs:  Placentitis, foetal death and abortion  Enteritis (diarrhoea)  Pneumonia
  • 10.  Polyserositis , polyarthritis  Lymphadenopathy  Keratoconjunctivitis  Meningoencephalitis  Epididymitis, orchitis, seminal vesiculitis TREATMENT: 1- Tetracycline antibiotic for 5 days 2- NSAID eg flunixin meglumin @1.1-2.2mg/kg I/M for 3 days 3- EYE OINTMENT eg. Polyfax eye oitment,tobramycin (tobradex eye drops) for 5 days. 4- FLUID THERAPY (RINGER LACTATE@150-200ml bid) Prevention: Oil-based, inactivated vaccine is produced by Onderstepoort Biological Products (OBP) to prevent abortions caused by chlamydial infection. It is essential to vaccinate before the breeding season, because the vaccine will not give protection against abortion once the foetus has been infected.(4 to 6 weeks before the breeding season) VITAMINS The fat-soluble vitamins are important in goat nutrition, as they are in the feeding of other domestic animals and humans. Water-soluble vitamins other than thiamine and niacin are generally ignored when formulating caprine rations. Vitamin A Vitamin A is a nearly colorless,fat soluble, long chain unsaturated alcohol with five double bonds. Beta-carotene is the standard dietary precursor of vitamin A, although some other carotenoids also have biological activity. Currently recommended conversions are 671 IU of vitamin A/mg of beta carotene and 436 IU/mg of other common carotenoids such as cryptoxanthine in yellow corn. As little as 10% of dietary beta-carotene escapes degradation in the rumen. Absorption, metabolism, and storage of the remainder require a healthy digestive epithelium and liver
  • 11. Vitamin A can also be expressed as retinol equivalents (RE), where 1 IU = 0.3 μg of retinol. The biopotency of 1 RE for goats is expressed as the vitamin A activity of 1 μg of all-trans retinol, 5 μg of all-trans beta carotene, or 7.6 μg of other carotenoids with vitamin A activity (NRC 2007). Signs of Defificiency Poor appetite weight loss, unthrifty appearance poor hair coat, night blindness thick nasal discharge in young animals,diarrheal and respiratory disease and to parasitism. To further compound the problem, kids with coccidiosis have an increased vitamin A requirement because of impaired absorption. Adult goats may have a decreased fertility rate, abortion occurs (related to inadequate steroid hormone synthesis) in addition to an increased susceptibility to disease. Vitamin A deficiency may promote desquamation of urinary epithelium and nidus formation with subsequent urolithiasis. PATHOGENESIS:
  • 12. Carotenoids are the principal source of vit A for small ruminants.The green part of plants are typically rich in carotene. The likelihood of a deficiency is minimal if small ruminants have access to modest amounts of green fodder. Alternatively,low carotene intake by grazing livestock is common in geographical locations that experience prolonged dry season & under condition of winter feeding may not occur more than 3-4 months.The circumstances conducive to vitamin deficiencies are extended periods of draught,diets primarily composed of grain based conncentrates mixture low in retinol equivalents neaonatal animals consuming milk from mothers on a low carotene content are substituted for yellow corn. The 1st stage was marked by the development of xerophathalmia which start with corneal opacity followed by ulceration of cornea and ended with the onset of corneal ulceration. The 2nd stage of deficiency was associated with a variety of symptoms starting with abnormal fluctuations in body weight followed by growth retardition and weight loss At about same time, weakness & incoordination of movement developed .Thereafter,convulsions were followed by death.Although clinical signs of deficiency were not observed in a long term Vit A depletion study. The hypovitaminotic A adult and a higher incidence of squamous metaplasia of the parotid duct and glands.Goats experiencing Vit A deficiency had reduced feed intakke and lostbody wt after 25 weeks. Vit A deficiency also decrease sperm quality,in pregnant goats they may abort or produce blind ,dead and weak offspering. Retained placenta case also exists. Dietary Recommendations and Supplementation The NRC applies sheep requirements to goats. According to its 2007 recommendations, the daily maintenance requirement is 31.4 RE/kg body weight or 104.7 IU/kg. The 50-kg goat at maintenance should receive 1,570 RE or approximately 5,235 IU of vitamin A/day while the 90-kg goat needs approximately 9,423 IU. To this is to be added 45.5 RE/kg (152 IU/ kg)/day for late pregnancy and 100 RE/kg (333 IU/ kg) for growing goats. The NRC 2007 additional requirement for lactation is now 53.5 RE/kg and dis regards the level of milk production. A simplifified recommendation from the French is 5,000 IU vitamin A/kg dry matter of feed across the board Browse and green leafy hays are good sources of vitamin A, while old or weathered hay is a poor source. After six months of storage, all beta-carotene in hay has been destroyed. Vitamin A palmitate is commonly added to mineral mixes and commercial concentrates. Because vitamin A is fat-soluble and stored in the liver and other body fat, the adult goat can tolerate several months of low carotene intake without developing clinical signs of deficiency. Vitamin A toxicity does not occur when natural diets are fed but could occur with mixing errors. A maximum of 6,000 μg of retinol/kg bodyweight/day is recommended (NRC 2007).
  • 13. Colostrum is a very rich source of vitamin A, and before its consumption kids have minimal stores. When kids receive colostrum from does that are primiparous (decreased concentration of vitamin A), or only limited amounts of colostrum, an oral vitamin A palmitate supplement is desirable. Injectable vitamin A is less valuable because of rapid peroxidation at the injection site. Vitamin E Vitamin E (1 mg dl-alpha tocopheryl acetate = 1 IU) is present in colostrum, milk, and many natural feeds, especially green forage. Ruminants do not synthesize vitamin E, but require it in their diet . Its main action is as an antioxidant; it stabilizes polyunsaturated fatty acids, vitamin A, and various hormones and enzymes. Vitamin E and selenium are closely interrelated; a defifi ciency of one can be at least partially offset by increasing the intake of the other. Signs of Deficiency Nutritional muscular dystrophy (white muscle disease). This is most likely to occur with feeding of silage or old hay, because the vitamin is lost in storage. Kids may have muscle disease at birth and be too weak to suckle. Sudden death related to Zenker’s necrosis of heart muscle or diaphragm may occur, even in kids with normal selenium status. Inhalation pneumonia can also result from muscle weakness of the larynx and pharynx.
  • 14. Reproduction failure, abortion in doe Affected kids are sometimes noted to cough or have milk run out the nose after drinking. Kids may also develop muscle stiffness after exercise. Adults may show poor uterine involution and retained placenta. Undesirable milk flavors related to oxidation of milk fat might be expected to occur with vitamin E/selenium deficiency. Ventral edema of Angora goats has also been associated with vitamin E deficiency. Vitamin E is also very important for optimizing immune responses. Beneficial effects have been demonstrated in ruminants on phagocytosis by polymorphonuclear cells and on cell mediated immunity. Obviously, increased incidence or severity of infectious diseases is a very nonspecifific sign of vitamin E deficiency, but is part of the reason for recent increases in the dietary recommendation. Plasma vitamin E concentrations of less than 1.5 μmol/l (65 μg/dl) in preparturient does and less than 1 μmol/l (43 μg/dl) in suckling kids have been associated with increased risk of myopathy . A normal range in goat serum of 60 to 150 μg/dl has been proposed by the Colorado Veteri nary Diagnostic Laboratory . Note that the blood sample needs to be handled very carefully (no hemolysis, rapid refrigeration and removal of the plasma or serum from the erythrocytes) if accurate results are to be obtained. Liver vitamin E concentration can also be determined and may better reflect the nutritional status of the animal. A normal caprine liver concentration of greater than 250 μg/100 g wet weight has been proposed. Dietary Recommendations and Supplementation In the absence of selenium defifi ciency, the daily requirement of vitamin E for preventing nutritional muscular dystrophy in preruminant lambs and calves, and presumably, kids was previously given as 0.1 to 0.3 IU/kg bw . This was to be doubled when feeding milk replacers. Later, it was advised to add 25 to 50 mg of vitamin E/kg of concen trate for adults and 50 to 100 mg/kg of concentrate for kids. Note that vitamin E is relatively nontoxic and 75 IU/kg bw daily is presumed safe. The current recommendation of 10 mg/kg bw daily takes into account the diverse functions of the vitamin beyond prevention of mypoathy. Vitamin E is oxidized by iron (or copper) in the feed, and thus diets with high iron content may have less vitamin E available. Rapidly growing green plants have a high content of polyunsaturated fatty acids (PUFAs) and animals on lush pasture also have a higher need for antioxidants such as vitamin E because
  • 15. of increased incorporation of PUFAs into cell membranes and thus increased susceptibility to lipid peroxidation . Oil seeds also contain high concentrations of PUFAs. The vitamin E content of colostrum depends on the nutrition of the dam during pregnancy. At the end of the winter, when most goats in temperate climates kid, the hay cut the previous year is very low in vitamin E. Supplementation of the pregnant doe is important for optimizing kid health. Some injectable preparations such as vitamin A and D include vitamin E as an antioxidant to stabilize the other fat-soluble vitamins; the quantity of vitamin E is inadequate for therapeutic purposes. Similarly, some vitamin E/selenium preparations contain relatively little vitamin E. Injectable preparations of vitamin E alone are available, with labeled recommendations of 600 to 900 IU to lambs at birth or weaning and 1,200 to 1,500 IU to ewes prepartum or at lambing. Magnesium Magnesium has received far less study than have calcium and phosphorus, although the metabolism of these minerals is interrelated. Approximately 62% of body magnesium is deposited in bone, 37% in cells, and 1% in extracellular fluid. Magnesium is required for many enzyme systems (including those necessary for energy metabolism and for synthesis of RNA and DNA) and for normal neuromuscular function. Signs of Deficiency: The normal serum magnesium concentration for goats is 2.8 to 3.6 mg/dl . Hypomagnesemic tetany typically occurs if the serum level drops below 1.1 mg/dl . Serum calcium may be low because magnesium is required for the release and action of parathyroid hormone. A magnesium deficiency may lead to anorexia hyperexcitability. decreased production, teeth grinding, salivation,
  • 16. tetany, seizures, recumbency, ABORTION Death, Goats can compensate somewhat for a dietary magnesium defificiency by decreasing output of both milk and urine. Dietary Recommendations and Supplementation The percent magnesium absorption probably varies considerably with the diet and is not regulated by a hormonal feedback system. Transit time that is too rapid or a potassium excess may hinder magnesium absorption. Grasses growing rapidly in cool, wet weather or after heavy fertilization are often low in magnesium and high in potassium. Forages containing less than 0.2% Mg on a dry matter intake (DM) basis have been associated with hypomagnesemic syndromes in ruminants. Whole goat milk also occasionally supplies inadequate magnesium, and kids might develop tetany if not supplemented For an animal that weighs 60 kg, then, total daily intake of magnesium is 1 g for maintenance (approximately 0.045 g/kg 0.75), plus 1.5 g for late gestation. The approximate magnesium supplement needed per kg of milk is 0.7 g. When additional magnesium is needed in the diet, it is often supplied at MgO, although MgCO3, MgSO4, and MgCl2 are other, although less palatable or more laxative possibilities. On spring pastures, provision of NaCl in loose or block form helps to counter high dietary potassium and maintain absorption of magnesium from the rumen. Copper and Molybdenum These two minerals are closely interrelated. High molybdenum (above 3 mg/kg) in the feed results in a relative copper deficiency, probably through formation of copper-molybdenum complexes in the tissues.
  • 17. Signs of Deficiency: nonspecific signs copper deficiency Inappetence, poor growth , weight loss, decreased milk production decreased cytochrome oxidase activity. Anemia occurs because ceruloplasmin is required to mobilize stored iron for synthesis of hemoglobin and myoglobin. Decoloration of the hair occurs because a copper-containing enzyme is necessary for melanin production. Swayback and enzootic ataxia in kids are related to defective myelination. Cardiac insufficiency is probably due to a combination of problems, including inadequate cytochrome oxidase activity and anemia. Osteoporosis and spontaneous bone fractures are also related to effects on copper-dependent enzymes. Abortions and stillbirths also occur. In addition, copper is required for proper function of the immune system. Dietary Recommendations and Supplementation Deficiency symptoms occur when dietary copper is less than 7 mg/kg and molybdenum is normal. A suitable level for ration formulation is 10 to 20 mg/kg DM , and it is generally recommended to keep the Cu : Mo ratio above 2 : 1 and below 10 : 1. Excessive calcium and sulfur both interfere with copper absorption as does excessive dietary iron. With feeding of corn silage or sulfur, dietary copper should be at least 14 mg/kg. Note that copper oxide is only one-third as digestible as copper sulfate. When the basal diet is deficient in copper or contains markedly excessive amounts of molybdenum, copper oxide wires administered orally in a gelatin capsule provide long-term (six-month) supplementation because they lodge in the abomasum and release copper slowly .
  • 18. An injectable supplement containing chelated copper is available in the United States labeled for goats. Copper Toxicity Adult goats are not as susceptible to copper toxicity as sheep , in part because of lower uptake by the liver. Liver copper stores are approximately ten times lower in normal goats than in sheep and cattle. In a toxicity study, hepatic copper concentrations were six to nine times higher in three-month-old lambs than kids. A deficiency of molybdenum (less than 0.1 mg/kg , then, does not generally induce copper toxicity but rather interferes with normal growth and fertility . Also, goats can safely consume a trace mineralized salt preparation formulated for cattle or a horse grain mix, whereas this is dangerous for sheep. Young kids, however, are sensitive to increased copper levels in the feed .One example of copper toxicity was reported in young Angora goat kids receiving milk replacer formulated for calves (10 mg/kg copper on a DM basis). The kids died of a copper-associated hemolytic crisis. It is likely that the preruminant kid absorbs copper more efficiently than does the adult goat. Fatal hepatic necrosis without hemolysis has also occurred in adult goats fed an improperly formulated mineral mix. Iodine In the absence of adequate iodine, the thyroid gland synthesizes an uniodinated inactive prehormone rather than thyroxine. In response to lower thyroxine levels the pituitary gland secretes thyroid stimulating hormone (TSH). As a result, the thyroid gland hypertrophies and produces the clinical condition goiter. Goats may produce kids with goiter on the same property where sheep and their lambs remain healthy. This is because the goats’ browsing habits result in less soil ingestion compared with the grazing sheep. Signs of Deficiency birth of weak or dead kids and a poor hair coat. Kids may appear “dumb” or unwilling to suckle. Growth rate of kids is reduced, as is the fertility of does. Dietary Recommendations and Supplementation The requirements of ruminants can usually be met by feeding 0.8 mg/kg iodine to lactating females and 0.5 mg/kg to the remainder of the herd. Cruciferous plants increase the iodine requirement to 2 mg/kg in the ration dry matter for lactation and 1.3 mg/kg for other animals. Iodized salt is a simple way to prevent defifi ciency, but it should not be force fed. A maximum tolerable dietary iodine level of 50 mg/kg has been established for cattle and sheep, with the proviso that the iodine concentration in the milk of animals on such a diet may be undesirable for humans . Owners should
  • 19. likewise be discouraged from feeding large quantities of kelp and other concentrated iodine supplements. Manganese Signs of manganese deficiency in goatsinclude reluctance to walk, deformed forelimbs (caused by defective cartilage formation), excessively straight hocks, reduced fertility (including silent estrus) or abortion in does Buck kids appear to show a greater depression in growth rate than doe kids when fed an extremely manganese-deficient diet (1.9 mg/kg). The recommended dietary level for goats by various authors has ranged from 20 to 120 mg/kg , but 60 mg/kg allows for interference with absorption, as by excess calcium. Selenium Selenium deficiency occurs when the soil in a locality is defifi cient (less than 0.5 mg Se/kg of soil) and locally harvested feeds are fed (less than 0.1 mg Se/kg of feed) . Selenium deficiency has occurred in animals and humans in many parts of the world, including the United States, China, Finland, New Zealand, and Australia. Signs of Deficiency: Many selenium deficiency signs are identical to those of vitamin E deficiency, as mentioned earlier. Nutritional muscular dystrophy, which can be caused by either vitamin E or selenium deficiency, Experimental selenium deficiency (less than 38 μg/kg DM) has produced lowered reproductive efficiency (apparent lowered conception rate) and decreased production of milk, milk fat, and milk protein in the following lactation. Selenoproteins act as antioxidants and are also involved in the conversion of T4 to T3. Increased supplementation with vitamin E masks a mild selenium deficiency. Potential selenium deficiency problems can be identified in clinically normal animals by evaluating their glutathione peroxidase status, because selenium is required for GSH-Px formation
  • 20. selenium deficiency can be suspected if the goat has less than 5 μg selenium/dl blood (less than 0.05 mg/kg). Serum selenium concentrations less than 0.05 mg/kg have also been deemed to indicate deficiency . Liver selenium content of animals that die or are slaughtered can be used to monitor the selenium status of the herd. Concentrations of 0.25 to 1.20 mg/kg wet weight are adequate, while concentrations of 0.01 to 0.10 mg/kg wet weight are deficient . Maternal liver stores of selenium are decreased in advanced pregnancy as transfer to the fetus occurs. When conversion of units is necessary for interpretation of laboratory reports, it helps to know that 1 μg selenium/dl is equivalent to 0.127 μmol/l . Dietary Recommendations and Supplementation Selenium should be present in the diet at a minimum of 0.1 mg/kg of feed. Sodium selenite and sodium selenate are permitted. Feed mills cannot add more than 0.3 mg/kg selenium to a complete ration for cattle or sheep or 90 mg/kg to a sheep salt-mineral mix, nor should the maximum daily intake of added selenium for sheep exceed 0.7 mg/head/day Supplementation of goat diets with selenium yeast is specifically permitted in the United States, with up to 0.3 mg/kg added selenium in this form allowed in complete feeds (FDA 2005). In the European Union, the maximum allowed selenium inclusion rate in the ruminant diet is 0.568 mg/kg, and feeding selenium yeast at ten times this level did not result in toxicity . When the soil, and hence the roughages and grains grown on it, are deficient in selenium, several methods have been used to improve selenium content of feeds for goats. One is to use sodium selenate in fertilizer mixes applied to the fields. In Finland, this practice has increased the selenium content of feeds from 0.02 mg/kg DM to 0.2 mg/kg DM . Deficiency symptoms are most likely to develop: These include shortly before breeding and four and/or six weeks before parturition for does, twice a year for bucks, and at birth and one month of age for kids. The dose administered is typically one to two times the labeled sheep dosage, with kids of normal size receiving the “minimum” dose at birth instead of two weeks of age. Because injectable selenium preparations available in the United States are not labeled for goats and are labeled as not for use in pregnant sheep, practitioners should be cautious about prescribing them for pregnant goats without informed owner consent. Selenium Toxicity
  • 21. There is a relatively narrow margin of safety with selenium; the maximum tolerable level in the feed of ruminants is currently estimated to be 5 mg/kg. Certain soils are termed “seleniferous” because of an increased selenium content, and certain indicator plants require and accumulate increased concentrations of the mineral. Some of these plants found in the United States are Stanleya, Haplopappus, and some species of Astragalus. They are very useful for indicating that the soil is dangerous, but they are not the only plants that accumulate toxic levels. Most crop plants,grasses, and weeds can accumulate as much as 50 mg/kg selenium when grown on seleniferous soils . . Acute selenium toxicosis (depression and dyspnea) has been produced experimentally in sheep with injections of 0.4 mg Se/kg bw, and the LD50 in this study was 0.7 mg Se/kg. Necropsy lesions included pulmonary edema and myocardial necrosis. Practitioners must guard against accidentally substituting an injectable selenium product marketed for adult cattle for the lower concentration calf and sheep product when small kids are treated. Selenium is less toxic when given orally. Daily oral doses of sodium selenite at 1 mg/kg body weight/day were nontoxic to growing Nubian goats, whereas a single dose of 40 mg/kg or two daily doses of 20 mg/kg were rapidly fatal. Adverse reactions (deaths and abortions) in several flocks of pregnant sheep have led to the relabeling of injectable selenium in the United States as not for use in pregnant sheep. These products are not approved for goats, and thus the practitioner who prescribes injectable selenium for pregnant goats may be at increased risk of legal action should any adverse reactions or unrelated abortions occur. PREGNANCYTOXEMIA: Excessive grain feeding itself can lead to pregnancy toxemia because the goat that eats inadequate rough age is apt to go off feed at this critical time. When goats are heavily fed on corn silage in late gestation they become obese. Their level of ingestion then decreases dramatically before parturition. Rumen acidosis from energy-rich silage may contribute to the development of pregnancy toxemia. Pathogenesis: Pregnancy toxemia is typically more common than lactational ketosis, and occurs predominantly in “improved” breeds with high prolififi cacy. It is not a disease expected to occur in native breeds carrying a single kid under extensive management conditions. With proper management and nutrition, even the doe carrying quadruplets can remain clinically healthy, although concentrations of ketone bodies in the blood can be expected to be higher in those goats carrying large litters.
  • 22. The developing fetuses depend upon glucose (maternal hepatic gluconeogenesis) for their energy needs. Ketone bodies and free fatty acids do not cross the placenta in any substantial quantities (Reid 1968). Insulin levels in the late pregnant doe are decreased; this spares glucose for fetal needs while at the same time stimulating lipolysis and gluconeogenesis. Placental lactogen levels are greatly increased when multiple fetuses are present . Placental lactogen has growth hormone as well as prolactin activity and is probably crucial to meeting the metabolic needs of the fetuses at the expense (if necessary) of the dam. Thus, the late pregnant doe is often subclinically ketotic. Clinical Signs The early signs of pregnancy toxemia are vague. They probably originate from decreased glucose utilization by the doe’s brain. The goat may be slow to get up or may lie off in a corner. It eats less and its eyes are dull. There is often a noticeable subcutaneous edema of the lower limbs. Teeth grinding and generalized weakness progress to more apparent neurologic abnormalities (blindness, loss of menace response, stargazing, nystagmus, ataxia, tremors), then to coma. The fecal output is reduced to a few small, dry, mucus coated pellets. As metabolic acidosis develops the animal may breathe more rapidly. Thus, advanced primary ketosis may be difficult to distinguish from a primary pneumonia which has caused the goat to go off feed and develop a secondary ketosis. A careful physical examination is also necessary to identify other problems such as parasites, lameness, and bad teeth that might have contributed to the animal’s present state.
  • 23. In the terminal stages of pregnancy toxemia, the doe becomes recumbent. Death of the fetuses at this stage releases toxins and hastens the demise of the doe. Pulse and respiratory rates increase as endotoxic shock develops. The course of the untreated disease varies from twelve hours to one week. Goats with pregnancy toxemia that do not die tend to have dystocias and higher kid mortality. They do not come to milk well. Similar problems are noted in obese goats, even when clinical pregnancy toxemia does not occur; in addition, these animals are at high risk of developing lactational ketosis. Laboratory Tests There are three major ketone bodies produced in the course of this metabolic disease: beta-hydroxybutyrate (BHB), acetoacetate, and acetone. In the past, these were sometimes measured together in a poorly defined way and reported as total ketones. Currently, BHB, which is the most stable ketone in blood and accounts for approximately 85% of the total ketones in sheep with pregnancy toxemia, receives the most attention in the laboratory. Some people can detect an odor of ketones on the breath of ketotic animals. Others must depend on simple diagnostic reagents or laboratory tests. In the early stages of pregnancy toxemia, ketone bodies are easily detectable in the urine. The commonly used test strips and pills containing nitropruside turn purple in the presence of acetoacetate but react minimally with acetone and BHB. If the doe has only a trace ketonuria (physiologic when late pregnant with multiple fetuses), some other cause for its illness should be sought, but supportive treatment to prevent worsening of the ketosis should be given. Late stages are usually accompanied by renal failure; marked proteinuria, epithelial casts, and ketonuria are present. The veterinarian should have a collection cup close at hand whenever examining a late-pregnant goat. BHB values less than 1 mmol/l can be considered normal, values of 1.5 to 3 mmol/l can be considered indicative of severe undernutrition, and animals with pregnancy toxemia often show a BHB concentration greater than 3 mmol/l. For conversion to other units, note that BHB in mmol/l × 10.3 = BHB in mg/dl. Necropsy Findings The doe that dies of pregnancy toxemia usually has multiple fetuses in the uterus, unless these were removed just before death. The fetuses may be fresh or
  • 24. decomposed. The doe’s liver is enlarged and yellow because of infifi ltration with fat . The doe’s adrenal glands are enlarged. The carcass appears dehydrated. If urine remains in the bladder, it shows a strong ketone reaction. Treatment The treatment and prognosis depend on the stage of the disease. In the earliest clinical form, the goat readily eats offered grain. Its diet should be improved to include better quality roughage and increased concentrates. Propylene glycol is given orally by dosing syringe, at the rate of 60 ml two or three times daily as a glucose precursor. Although some authors suggest as much as 175 to 250 ml of propylene glycol twice a day , this dosage seems excessive and likely to overwhelm the ability of the rumen flora of an already sick goat to digest it. A commercial product that contains niacin as well as propylene glycol has been recommended, or the goat can be injected with enough mixed B vitamins to supply 1 gram of niacin/day. Calcium borogluconate (60 ml of a 23% to 25% solution) is given subcutaneously to counteract any concurrent hypocalcemia; approximately 20% of sheep withpregnancy toxemia are also hypocalcemic. If the animal is unwilling to eat or to rise, the prognosis is guarded. Intravenous glucose (25 to 50 grams, preferably as a 5% to 10% solution), mixed B vitamins, and force feeding are added to the regimen. If the goat is known to be within one week of its due date, hormonal induction of parturition with 10 mg of prostaglandin F2 alpha will end the energy drain to the fetuses. If the due date is uncertain and the owner desires to save doe and kids, 20 to 25 mg dexamethasone may be preferred for its gluconeogenic effects and beneficial stimulus to appetite. Prevention Animals must be fed high quality rough age and as much as 500 g of concentrate daily. Any conditions that disturb the comfort of the goat, such as lack of exercise, poor ventilation, or drafts, should be corrected. This means that the stall should be dry, well bedded, and uncrowded. The goats should be let loose for at least two to three hours per day.
  • 25. Goats carrying three or more fetuses should receive the best quality roughage available in addition to adequate concentrate. When one doe develops pregnancy toxemia, the diet of the rest of the herd must be evaluated and corrected as necessary. Concentrates should be introduced gradually and under strict control, to avoid indigestion. It is unrealistic to expect a large herd of commercial goats to be totally free of pregnancy toxemia. Routine monitoring of all late-pregnant does for urinary ketones is also unrealistic. Healthy does carrying large litters can be expected to excrete small quantities of ketones yet do not need treatment. Drenching with prophylactic propylene glycol (60 ml orally twice a day) is in itself a stress to a goat and should be reserved for those showing abnormal behavior or diminished appetite. Bluetongue in Goats
  • 26. A severe viral disease caused by an orbivirus transmitted mainly by gnats of the genus Culicoides. Transmission sexually and across the placenta can also occur. Because the vector is a gnat, the spread of this disease occurs primarily in the late summer and fall. The virus is endemic in many areas and cattle and wild ruminants, or white-tailed deer, act as reservoirs. Goats are commonly infected with the virus but rarely show any signs of clinical disease; it is a self-limiting disease in goats. Symptoms: transient fever swelling of the face, muzzle, and ears; large amount of nasal discharge which may cause crusting around the nose; oral mucus membranes become dark pink small hemorrhages and ulcers may form on the roof and corners of the mouth. The tongue may become cyanotic (blue) but not as common as the name indicates. Laminitis can develop caused by inflammation of the coronary band and tissues of the foot to the point that some animals may slough their hooves. Diarrhea and wool-break will also occur in infected animals. Bluetongue virus will cause abortions, stillbirths and weak lambs. Diagnosis: By the presence of clinical signs similar to those reported in sheep have been documented in goats. Treatment: Minimize animal stress antibiotic treatment for secondary infections I.e amoxicillin for 3 days.
  • 27. Prevention: Controlling breeding areas for biting gnats. Keeping animals away from areas where biting gnats are present. Vaccine is available for sheep. Listeriosis Listeriosis is an important infectious disease of goats most commonly associated with neurologic disease, but also capable of causing septicemia and abortion. The organism can be shed in the milk of healthy-appearing carrier goats as well as in the milk of sick goats. The zoonotic potential of listeriosis from milk and dairy products is a growing concern . Etiology Listeria monocytogenes is a motile, aerobic and facultative anaerobic, small, Gram-positive rod. It produces a narrow zone of beta hemolysis on blood agar. It is capable of growth over a wide pH range of 5.5 to 9.6 and a temperature range of 37.5°F to 113°F (3°C to 45°C), but optimal growth occurs at pH 7 to 7.2 and a temperature range of 68°F to 104°F (20°C to 40°C). Isolation of the organism from tissues and organic materials such as animal feeds can be difficult, so dispersion of tissues in a blender, cold enrichment of samples, subculturing from tryptose phosphate enrichment broth, and use of selective media such as trypaflavine nalidixic acid serum agar have been recommended. L. ivanovii is considered non-pathogenic to humans but is pathogenic for mice and has been associated with abortions in sheep and cattle. In one recent report, abortions were reported in sheep in a mixed sheep and goat flfl ock, but the goats were not affected. Though easily killed by common disinfectants, L. monocytogenes can survive in feces, silage, and tissue for fifi ve or more years There are now sixteen known serotypes with numerous subtypes. Serotype 4, especially type 4b, and to a lesser extent serotype 1 have been associated with encephalitis and septicemia in goats. Abortion is associated primarily with serotype 1 . In one outbreak of listeriosis in goats, the same serotype, 4b, was recovered from goats with encephalitis and from goats with abortion. It was hypothesized that transmission in this herd outbreak was by the venereal route. Serotypes 1/2a, 1/2b, and 4b are the serotypes most commonly isolated from human cases of listeriosis and from livestock cases.Because L. monocytogenes is widely distributed in nature, characterization of environmental isolates by serotyping or phylogenetic analysis is necessary to confirm their association with disease outbreaks.
  • 28. Epidemiology As many as forty species of birds and mammals, including humans, can be infected with L. monocyto genes and the organism has been isolated on six continents. Listeriosis is a well known, sporadic clinical problem in intensively managed dairy goats in North America and Europe. In France, 4.9% of fecal samples from sheep and goats in ninety-eight flocks yielded L. monocytogenes. A seroepidemio logic study in Spain identififi ed infection in 5% of goat herds tested. Caprine listriosis has also been reported from Japan , South Africa , Australia ), India , Brazil , and Turkey . Factors predisposing to clinical listeriosis in goats are similar to those reported for other farm animals and include sudden changes in weather, feeding regimens, or general management procedures; confinement in winter, particularly if overcrowded and with poor sanitation; increased stress from poor nutrition, parasitism, or other concurrent disease; advanced pregnancy; and the feeding of silage, particularly poor quality silage. The feeding of silage is often empha sized as a key predisposing factor in ruminant listerio sis (Morin 2004). However, a history of silage feeding is not a prerequisite in outbreaks of caprine listeriosis An increased occurrence of listeriosis in fall and winter has also been observed in goats, though cases can occur year-round. The disease is most common in adult goats. Based on experimental challenge studies, goats are more susceptible to L. monocytogenes infection than sheep. A survey from Greece based on microbial cultures of brains from animals with neurologic signs of disease also found that goat herds were affected with encephalitic listeriosis more frequently than sheep flfl ocks, with serotype 4b being predominant . The source of infection in herds is not always clear. Recent studies have identififi ed some differences in the ecology and transmission of L. monocytogenes on cattle farms as compared to sheep and goat farms in upstate New York. Cattle farms had a higher level of environmental contamination with L. monocytogenes than small ruminant farms whether the cattle had a history of clinical listeriosis (case farms) or not (control farms). When small ruminant case farms were compared with bovine case farms, isolation of L. monocytogenes in small ruminant fecal samples was signififi cantly less common than in bovine fecal samples. However, the organism was signififi cantly more common in feed samples from small ruminant case farms than from bovine case farms, indicating that listeriosis on goat and sheep farms is more likely transmitted via feed than feces. On all farms, soil samples were positive more commonly than were feed samples, indicating that soil is an important source of feed contamination with L. monocytogenes. Direct transmission of L. monocytogenes from animals to humans can occur, it is uncommon. In such cases human symptoms are usually limited to localized cutaneous infections. Much more common is the transmission of the infection to humans via
  • 29. foods of animal origin. Zoonotic infection from goat milk and goat milk products is a very real concern. Listeria mono cytogenes can be shed in the milk of clinically affected goats as well as normal-appearing latent carriers. Shedding is less likely in the encephalitic form of the disease than in the septicemic or abortion forms. In latent carriers, the intensity of shedding is increased toward the end of gestation ). Listeria monocytogenes has resisted pasteurization at 143°F (61.7°C) for thirty-five minutes, but is killed by high temperature short-time pasteurization at 160.9°F (71.6°C) for fifi fteen seconds. The intraleukocytic location of some of the organisms in milk presumably contributes to this pasteurization resistance (Blenden et al. 1987). Experimentally, the organism has been reisolated from semi-soft, aged, goat milk cheeses made from unpasteurized, L. monocytogenes-inoculated goat milk as long as eighteen weeks after preparation . The organism has been isolated from retailed, pasteurized, flfl uid goat milk in the UK (Roy 1988). In a study in Sri Lanka, L. monocytogenes was recovered from raw goat milk, standard pasteurized milk, and cheese, but not from sterilized milk, ultra-high tem perature (UHT) milk, yogurt, or curd. Pathogenesis In the encephalitic form of listeriosis, the organism gains entrance to nerve endings in the oral cavity via breaks in the oral mucosa caused by coarse food, dental abrasions, or the loss of deciduous teeth. It then migrates up the nerves to the brain stem, where it stimulates a localized inflammatory response in the form of microabscesses comprised primarily of neutrophils. It is believed that L.monocytogenes primarily induces a cell-mediated immune response in the host and the severity of the resulting lesions may be mediated by the degree of immune recognition of the organ ism. Microabscesses are most common in the medulla and lead to destruction of cranial nerve nuclei, notably nerves V through IX. The cranial nerve defifi cits seen clinically reflfl ect this process. Occasionally, generalized meningitis can occur in addition to focal encephalitis. The incubation period in the encephalitic form may be two to three weeks. In the septicemic form, the incubation period may be as short as one day. The organism is believed to gain entry through the intestinal mucosa. There is an initial bacteremia with fever. This may be followed by recov ery, development of a latent carrier state, or progression to more severe clinical disease. Because themorbidity rate is often low in outbreaks of septicemic listeriosis, it is presumed that many animals handle transient bacteremia effectively and are only subclinically infected. When animals do become ill they may die within forty-eight hours or the illness may last for several weeks. Pregnant does abort several days after the initial fever and aborted fetuses also show evidence of septicemia. Septicemic goats may excrete the organism in feces and milk during and after clinical illness. Newborn kids exposed to the colostrum or milk of infected does can show signs of septicemia in the first few days of life.
  • 30. Seroconversion is marked in goats after septicemic listeriosis but mild in goats after encephalitic listeriosis . Ocular forms of listeriosis are also reported in cattle and sheep. Keratoconjunctivitis and iritis appears to result from direct contact of the eye with Listeria present in silage during the act of feeding (“silage eye”). Clinical Findings The encephalitic form is the most common in goats. Though unusual, septicemic and encephalitic listerio sis have been reported in the same goat herd The initial signs of the encephalitic form are non specififi c and include depression, decreased appetite, decrease in milk production transient fever of up to 107.6°F (42°C). These prodromal signs may be followed by incoordination hemiparesis with a tendency for the goat to lean, stumble, or move in one direction only. This tendency progresses to obvious torticollis and circling in the same direction. In advanced cases, the goat may be recumbent with the head pulled tightly into the flank, unable to straighten the neck voluntarily. Deficits of the facial nerve are also common and may occur with or without concurrent hemiparesis and circling. The signs are usually unilateral and include ear droop, ptosis, flaccid buccal muscles with accumulation of feed in the buccal pouch, salivation, and a collapsed nostril . Slack jaw, weak tongue, impaired swallowing, and nystagmus may also be seen. When lesions are bilateral, some of these deficits may be paradoxically less obvious since the abnormalities are symmetrical. Keratitis may be observed as a sequela to abnormal eyelid function. Loss of excessive saliva through drooling and the inability to swallow can lead to acid-base imbalance, electrolyte and flfl uid losses, dehydration, and weakness. The course of encephalitic listeriosis in goats is usually one to four days, which is shorter than that observed in cattle. The morbidity rate is variable, but the mortality rate can be high. The septicemic form also begins with depression, loss of appetite,
  • 31. decreased milk production , and fever up to 107.6°F (42°C). In these cases, fever may persist and the animal grows progressively weaker over the next several days. Neurologic signs rarely develop, but diarrhea, often bloody, is a common finding in goats. Goats may die within a few days or remain ill for several weeks. Pregnant does abort several days after the onset of septicemia. Clinical Pathology and Necropsy The hemogram may remain normal, particularly in the encephalitic form of the disease, or show a neutrophilic leukocytosis. The monocytosis seen in laboratory animals does not occur in ruminants. Analysis of the cerebrospinal flfl uid may be helpful. Protein levels and cell counts are typically moderately elevated. The cells are predominantly monocytes and lymphocytes, with some neutrophils also present. Bacteria are rarely seen in the CSF, and culture from the CSF is almost always unrewarding. Historically, serology, while useful for epidemiologic studies, has not been widely applied for diagno sis of individual cases .
  • 32. A number of different serodiagnostic techniques have been employed using crude antigens, but a general limitation for all these techniques was a lack of specificity, with cross reactions to other Gram-positive organisms being common . Another practical limitation is that in cases of encephalitic listeriosis, affected animals do not appear to mount a consistent, detectable humoral immune response, in contrast to cases of septicemic listeriosis. An increase in indirect hemagglutination titer is reported after septicemic listeriosis, but not after the encephalitic form . Sero-agglutination was found to be unsatisfactory as a screening test for accurately identifying herds with enzootic listeriosis. ELISA tests have been developed in recent years to detect antibodies against a specififi c antigen, listerioly sin O, which is an extra-cellular 58 kDa haemolysin, produced by all the pathogenic strains of L. monocytogenes Gross post mortem findings are uncommon in the encephalitic form of the disease, though visible, focal gray discoloration and malacia of the brain stem have been observed in affected goats . In the septicemic form, multiple foci of necrosis may be seen in liver, spleen, kidney, and heart. Multiple, small yellowish spots on the liver of aborted fetuses are highly suggestive of listeriosis. Placentitis and endometritis may also be observed in does that abort. Successful culture is most likely from the liver, spleen, lung, and uterus of septicemic adults. Diagnosis Neurologic diseases that can produce localizing signs consistent with a diagnosis of listeriosis include the neurologic form of CAE, focal brain abscesses, cerebrospinal nematodiasis, coenurosis, middle ear infections, bacterial meningitis, early rabies, and trauma to the facial nerve. The differential diagnosis for septicemic listeriosis, particularly when diarrhea is present, includes salmonellosis, yersiniosis, and enterotoxemia. When weakness predominates and diarrhea is absent, milk fever, and pregnancy toxemia should be ruled out. Treatment Penicillins, tetracyclines, and where permitted, chloramphenicol, are effective antibiotics.
  • 33. Adult goats with the septicemic form of disease responded favorably to intramuscular penicillin administered for three consecutive days at a dose of 2.5 g per day, but shorter courses of therapy were less effective. In the encephalitic form, intravenous sodium penicillin at a dose of 40,000 IU/kg every six hours until improvement is noted, followed by a seven-day course of intramuscular procaine penicillin at a dose of 20,000 IU/kg twice a day has been recommended . Oxytetracycline should be given intravenously at a dose of 10 mg/kg twice a day for at least three days. These high dosage levels are necessary to promote passage of antibiotic across the blood brain barrier and development of high-tissue concentrations in the CNS. It has been reported that ampicillin or amoxicillin given in conjunction with gentamicin is the treatment regimen of choice in human listeriosis cases. The use of a combination of gentamicin given at a dose of 3 mg/kg bw IV BID and amoxicillin given at a dose of 7 mg/kg bw IM BID was reported in one retrospective case study in small ruminants . The outcomes for sheep and goats treated with gentamicin/ampicillin were better than those treated with either penicillin or oxytetracycline. However, more of the animals treated with gentamicin/amoxicillin had a favorable prognosis at the onset of therapy because they were not yet recumbent. The use of gentamicin in goats is problematic due to prolonged antibiotic residues in meat and milk. Dexamethasone given once a day at a dose of 0.1 mg/kg intravenously has also been used in con junction with antibiotics in the treatment of encephalitic listeriosis with the rationale that steroids may suppress the infifi ltration of mononuclear cells that lead to microabscesses in the brain stem. The nonsteroidal anti-inflfl ammatory drug flunixin meglumine has been used at a dose of 2.2 mg/kg bw IV SID in the treatment of goats with encephalitic listeriosis but its contribution to a favorable outcome is not documented. Supportive therapy in the form of fluid and electrolyte administration, supplemental feeding, and management of exposure keratitis associated with lid paralysis may be necessary in severely affected animals. Large amounts of bicarbonate and fluid may be lost when salivation is prolonged and intense so fluid therapy should be tailored to address these deficiencies. Control In outbreaks of disease, aborting does should be isolated from the herd, and kids should be raised separately from adults.
  • 34. Aborted fetuses, placentas, and discharges should be handled wearing gloves and face masks, and disposed of carefully. Kids should not receive unpasteurized colostrum or milk from does involved in the outbreak to avoid neonatal septicemia. Because of the zoonotic potential, no unpasteurized milk should be consumed from goats in a herd with a history of listeriosis since shedding of the organism in the milk of latent carrier animals does occur. A vaccine has been in use in central Europe and Norway to protect sheep from listeriosis. While case rates in vaccinated and unvaccinated sheep were similar, the severity of disease was less and the response to treatment better in vaccinated animals Goat farmers who produce or market milk, cheese, or other dairy products need to be aware of the zoonotic potential of these products if contaminated with L. monocytogenes. There should be no sale of raw milk products from herds where the Listeria infection status in not known to be negative. Some of the challenges of keeping cheeses free of listeria contamination in small scale cheese making operations and approaches to reducing the risk of contamination of soft goat cheeses have been reported. Johne’s disease / Paratuberculosis Paratuberculosis, also known as Johne’s disease, is an economically important infectious disease of domestic and wild ruminant animals primarily affecting the digestive tract. Infection leads to gradual debilitation and death through a mechanism of digestive dysfunction that is not completely understood. Because the disease has been studied most intensively in cattle, many aspects of the bovine disease traditionally have been assumed to hold true for goats. However, this is not the case. Diarrhea, which is the cardinal sign in cattle, is an uncommon clinical sign in goats. Caprine paratuberculosis is characterized mainly by chronic,progressive weight loss in adults. Etiology
  • 35. Paratuberculosis is caused by the bacterium for merly known as Mycobacterium paratuberculosis (also previously called M. johnei). Based on advances in the molecular characterization of the organism that occurred in the 1990s, the organism has since been reclassified as a subspecies of Mycobacterium avium and is referred to as M. avium subsp. paratuberculosis (Map). Analysis of Map strains from different geographic locations and different host species using various molecular techniques indicates that there are two main groups of strains. The C, or cattle, strains occur mainly in cattle but are also the predominant strains in goats. The C strains also occur in deer and, rarely, sheep. The S, or sheep, strains occur mainly in sheep but also in farmed deer and sometimes goats. The presence of S strains in goats, though less frequent than the C strains, occurs mainly when goats are commingled with infected sheep. Restriction endonuclease analysis and DNA hybridization studies indicate that there may be a unique Map strain that is isolated from Norwegian goats that falls neither in the C nor S strain groups, but other studies could not distinguish Norwegian goat strains from cattle strains. Map is a small mycobacterium (0.5 × 1 microns) compared with other pathogenic mycobacteria. It demonstrates typical acid fastness with Ziehl-Neelsen stain. In tissues, the organism tends to be found in clumps inside macrophages rather than as individual bacteria. The bacterium is very resistant to environmental degradation and can persist in barnyards and in manure spread on pastures for longer than one year. Shade on pastures prolongs the survival time and thereare indications that Map may be genetically capable of dormancy, i.e., being able to enter a viable but non cultivable state and later reverting to a vegetative form . Disinfectants capable of eliminating the organism from the environment include cresylic compounds diluted 1 : 64 and sodium ortho phenylphenate diluted 1 : 200. The C strain of Map is a fastidious, slow-growing organism in vitro. Positive cultures are rarely identififiable before six weeks and should be held at least twelve weeks for confirmation of negative cultures. The cultivation of fifi eld isolates of Map requires supplementation of the culture media with mycobactin, an iron-chelating substance found in other Mycobacterium spp. Epidemiology Paratuberculosis was first definitively described in cattle in 1895 and in goats in 1916 ). It has traditionally been considered a disease of temperate regions with sporadic occurrence in tropical environments, primarily as a result of importation of infected livestock from endemic areas. Today, paratuberculosis is generally considered a disease of worldwide distribution. However, accurate details on the geographic distribution of caprine paratuberculosis are diffcult to ascertain, because statistics on goat disease are often reported together with sheep by governments and international agencies.
  • 36. Caprine paratuberculosis has been specifically described in numerous countries representing all continents but Antarctica. These countries include the Sudan in Africa; India , Nepal, and Korea in Asia; Turkey, and Israel in the Middle East; Cyprus , France , Greece, Norway), Spain , and Switzerland in Europe; Canada, the United States , and Mexico in North America; Chilein South America; and Australia and New Zealand . This also suggested that paratuberculosis might be a zoonotic disease, potentially transferable by contact with infected livestock or by consumption of food products from those animals. A comprehensive study by the National Research Council published in 2003 concluded that “there remains insuffifi cient evidence to prove or disprove that Mycobacterium avium subsp. paratuberculosis is a cause of some or all cases of Crohn’s disease in humans” . However, it was also concluded that a causal link between paratuberculosis and Crohn’s is plausible and a new research approach is needed to defifi nitively establish or disprove that relationship. Nevertheless, there are members of the human medical community who believe quite strongly that suffifi cient evidence exists to make that link and they call for a strong and immediate public health response . Pathogenesis The primary mode of transmission of paratuberculosis is fecal-oral, with the organism shed in the manure of infected adults and ingested by susceptible young stock, particularly when animals are overcrowded and sanitation is poor. Neonates are considered most susceptible to new infections, particularly when their own dam is an active fecal shedder and the kid is allowed to remain with the doe. However, age-related resistance is not absolute, and it is probable that adult animals may remain at risk for new infection if kept in overcrowded, heavily contaminated environments. The role of in utero transmission in the goat remains unknown, although the organism has been recovered from the uterus and fetal organs of some experimentally infected goats. If fetal infections were shown to occur naturally in goats, then currently accepted disease control practices would require modification. As with cattle and sheep, goats are most likely infected at an early age by ingestion of the organism. Following ingestion, the organism localizes in the mucosa of the small intestine and associated lymph nodes. The organism is transported across the intestinal mucosa by M cells associated with the Peyer’s patches or by enterocytes While some exposed individuals may develop resistance to chronic infection, many infected goats subsequently carry the infection in a dormant state in the Peyer’s patches of the intestine and the mesenteric lymph nodes for a variable period into adulthood. At some point, triggered by stress or other ill-defined factors, some infected animals begin to shed the organism in the feces. They may begin to show clinical signs concurrently or at a later time.
  • 37. In general, small ruminants with paratuberculosis tend to have less severe lesions of granulomatous enteritis than affected cattle. There have been a number of recent studies of experimental paratuberculosis infection in goats aimed at gaining a better under standing of the pathogenesis of the disease at a cellular level and the associated cell mediated and humoral immune responses . A standard experimental challenge model for caprine paratuberculosis has been proposed. There is experimental evidence that bacteremia occurs in goats with clinical paratuberculosis based on isolation of the organism in blood cultures and numer ous tissues at necropsy, including the udder and uterus . This suggests that offspring born to clinically infected does have a very high likelihood of infection through the birth process or when suckling, if not already infected as fetuses. Because of the prolonged dormancy of infection, the persistence of the organism in the environment, and the endemic nature of the disease, all goats in a known infected herd must be considered at risk of infection. Goats in an infected herd may fall into one of four categories: resistant or non-infected individuals; infected, non-shedders; subclinically infected, inap parent-shedders; and clinically affected, apparent shedders. Only individuals in the last group can be identififi ed as abnormal on physical examination, although a defifi nitive diagnosis can never be made on physical examination alone. The subclinical infection rate in an infected herd is likely to be much higher than the rate of clinically apparent cases. Clinical Findings Overt clinical disease rarely occurs before one year of age and is most common in goats two and three years of age. Clinical disease is often triggered by some episode of stress, such as parturition or recent introduction into a new herd. Affected individuals begin a course of progressive weight loss, which may extend from weeks to months and can lead to dramatic emaciation . Appetite may remain intact initially, but decrease later on, and the animal becomes increasingly lethargic and depressed. A rough hair coat and flaky skin are common. Animals in the advanced state of disease may become immunologically anergic. Advanced cases eventually succumb to debilitation, inanition, exposure, or secondary infections. Unlike cattle, goats rarely show persistent watery diarrhea, except possibly in the terminal stages of disease. As the disease progresses a moderate anemia of chronic infection may develop and clinical evidence of hypoalbuminemia such as intermandibular edema may be seen. Clinical Pathology and Necropsy Hypergammaglobulinemia and hypocalcemia have also been reported in affected goats . Definitive diagnosis of paratuberculosis requires bacteriologic or serologic testing and/or histopathologic examination of tissues. Infection with paratuberculosis slowly evolves in such a way that no single diagnostic test can possibly detect all infected animals at any given time in the
  • 38. course of the disease. The IFNγ assay was used to monitor the progression of paratuberculosis in a naturally infected herd of pygmy goats along with fecal culture, ELISA, and AGID. The IFNγ assay produced both false positive (one of three) and false negative (three of ten) results in live animals based on the true infection status determined subsequently by necropsy and culture of Map from tissues . In experimentally challenged goats, CMI was measured by IFNγ assay, but also by interleukin 2 receptor expression and by lymphocyte proliferation test. CMI responses were detectable by nine weeks post inoculation and persisted with variation through the two years of the study, though responses were stronger in the first year. The tests currently in use for identification of antibody responses in goats are the complement fixation (CF) test, the agar gel immunodiffusion (AGID) test, and the ELISA test. In general, when applied to goats, these tests have a sensitivity in the range of 85% to 100% when applied to clinical cases but only 20% to 50% when applied to subclinical cases (Stehman 2000). Molecular probes and PCR techniques have also been developed to confirm the presence of Map-specific genetic material in feces or tissue samples, in an effort to bypass the need for culture. However, inhibitors in feces and inefficient DNA release from clinical samples have hampered the general application of these techniques, particularly with regard to fecal samples. Most laboratories with the capacity to culture Map still offer the conventional solid media technique as the main or sole method for detection of Map in feces. Diagnosis The differential diagnosis of paratuberculosis in goats is essentially the differential diagnosis of chronic weight loss. Treatment There are no known effective treatments for the elimination of Map infection and none are specififi cally approved for the treatment of paratuberculosis. Various antimycobacterial drugs have been used with limited effect in goats. Attempted unsuccessful therapies have included isoniazid, isoniazid plus rifampin, isoniazid plus ethambutol, or all three drugs together . However, some treatment regimens have alleviated clinical signs in individual cases as long as therapy is continued.y. One such regimen is daily streptomycin sulfate (0.5 g intramuscularly) plus isoniazid (25 mg by mouth) plus sodium aminosalicylate (850 mg by mouth) for six months. A second regimen is daily dihydrostreptomycin (0.5 g intramuscularly) plus rifampin (300 mg by mouth) and isoniazid (300 mg by mouth) twice daily.
  • 39. Treatment has little economic application in goats except for the possibility of keeping known infected, high-quality, breeding does in good health long enough to achieve embryo transfers or to prolong the life of pet goats. Control The second major stimulus for control of paratuberculosis is the much publicized zoonotic potential of the disease relative to Crohn’s disease in humans, which has caused concern among public health officials, regulatory veterinary agencies, and livestock producers about public perception of milk and meat as whole some foods. As a result, numerous countries have initiated or expanded their paratuberculosis control programs in recent years. Most programs focus on cattle, but some may include sheep and goats. Improved management and sanitation help reduce the incidence of new infections in young stock. In intensive dairy operations, kidding pens should be cleaned or rebedded between kiddings. Kids should be taken immediately from the does and raised in separate facilities on heat-treated colostrum and pasteurized milk or milk replacer. Kids should not commingle again with adult animals until they themselves have kidded. Adult animals should not be over crowded, manure should be removed frequently from the pens, and feed and water should be provided in a manner that does not allow for fecal contamination. If manure is spread on pastures, goats should not be allowed to graze those pastures for at least one year. The use of pastures in general may be problematic as it has become clear that wildlife reservoirs exist for Map which may contaminate pastures, even when paratuberculous livestock have been kept off those pastures. Toxoplasmosis Etiology The protozoan Toxoplasma gondii is a very important cause of abortion, mummifification, stillbirth, and birth of weak young in goats as it is in sheep.
  • 40. Epidemiology and Pathogenesis. Cats serve as the defifi nitive host for this parasite they become infected by consuming uncooked meat scraps, placentas, and small rodents . Recently infected cats then shed oocysts in their feces. Oocyst shedding typically lasts from three to nineteen days, but oocysts may persist in moist and shaded soil for as long as eighteen months. Goats become infected by eating grass, hay, or grain contaminated by cat feces. After initial invasion of the goat’s small intestine and associated lymph nodes, the Toxoplasma organisms spread via the bloodstream to other tissues, including muscle, brain, and liver. Here the parasite may remain encysted for months or even the life of the goat. If the goat is pregnant at the time of initial infection, Toxoplasma commonly invade the placenta and fetus approximately two weeks after initial infection of the doe. Fetuses infected in the first half of pregnancy are more apt to die than fetuses infected in the second half. Sometimes abortion is repeated in the next gestation , but previously infected goats are usually resistant to abortion or other clinical signs when challenged with T. gondii Diagnosis Fetal serology is a very specififi c test for abortive toxoplasmosis . The ovine fetus (and presumably the caprine fetus) begins to develop immunocompetency at sixty to seventy days of gestation. First immunoglobin M is produced, but by ninety days immunoglobin G is being synthesized. Thus, if the fetus is infected after midgestation, it will usually produce antibody detectable by a variety of immunologic tests. A modified direct agglutination test (MAT) is considered to be very sensitive and can be used on any species, including goats, because species-specififi c conjugates are not used . Heart blood or thoracic flfl uid can be harvested from the fetus. Fetal autolysis does not always preclude identification of toxoplasma antibodies. Absence of antibodies in the fetus does not rule out toxoplasmosis because the fetus may have been infected too young or antibodies may have decomposed. Screening at both high and low dilutions is recommended to avoid false negatives when the antibody concentration is so high as to occupy all binding sites.
  • 41. Serologic testing of the doe at time of abortion is useful. Absence of antibodies is considered to be conclusive evidence that toxoplasmosis was not the cause of abortion . If the diagnosis is to be made by histology, it is very important that placenta be submitted. Small yellowish-white foci of mineralization confined to cotyledons are apparent grossly if abortion is delayed until forty-five days or more after infection. Washing the cotyledons thoroughly in isotonic saline solution makes deeper foci easier to visualize. Another useful technique is to compress the cotyledon with a glass microscope slide; the mineralized foci resist squashing. Microscopic foci of necrosis can be identififi ed after thirty days; tachyzooites are sparse in these lesions and diffifi cult to locate (Dubey 1988). Nonsuppurative encephalomyelitis is found more consistently than myocarditis, but even under ideal experimental conditions it is diffifi cult to fifi nd T. gondii organisms in tissue sections. Prevention To prevent exposure of susceptible goats to the oocysts in cat feces during the period of danger, which is pregnancy. In particular, grain should be stored in covered containers and the mangers kept clean. Contamination of the hay supply (by cats living in the hay barn) has been implicated in several outbreaks . If possible, feed the hay off the top of the stack to the nonpregnant does and young stock. A vasectomized tomcat might be helpful in keeping stray cats off the farm, but this technique has not been evaluated. Raw meat should not be fed to cats A live toxoplasmosis vaccine available for sheep in the UK might be effective in goats . Repeated administration of killed vaccine or vaccination with a related but nonpathogenic organism (Hammondia) might be effective in preventing abortion but would certainly be expensive; such vaccines are unlikely to be marketed in the near future. When abortions caused by toxoplasmosis are diagnosed, emphasis should be put on properly disposing fetuses and placentas, wearing protective gloves when handling these items, and properly pasteurizing milk and cooking meat. Pregnant women should be especially careful.